Sie sind auf Seite 1von 67

1.

Phentolam ine is used in the managem ent of


A. status epilepticus
diazepam 10-20 mg; lorazepam 4m g then phenytoin 1.5 gm
Kids get Valium at 0.3 mg/kg or Ativan at 0.1 m g/kg
B. cerebral edem a
M annitol 12.5 to 25 gm , crystals, reheat; decadron 6-10 mg
C. pheochrom ocytom a
sm all tum ors of adrenal gland that increase blood pressure due
to release of epi and norepi; Regitine is an alpha blocker
D. Cushing's Disease
am inoglutethim ide or m etyrapone; m itotane and ketoconazole
m ore potent and m ore ADRs; m ifepristone blocks corticosteroid receptors
E. hem orrhage
am inocaproic acid, Prem arin Inj, platelets, fresh frozen plasm a

2. W hich of these clotting factors is found in the extrinsic clotting system?


A. Factor I
B. Factor II
Heparin 2, 9, 10 via antithrom bin III; plasmin breaks up fibrin; PTT
C. Factor VII
W ararin 2, 7, 9 and 10 (vitam in K, green leafy vegetables); PT to calc INR
D. Factor IX
INR should be 2.0 to 3.0 except with m echanical heart valve (2.5 to 3.5)
E. Factor X
increase/decrease dose by 20-25% when adjusting
3. Kernicterus is a drug-induced disorder that m ay occur in neonates treated with
A. isoniazid
B. sulfisoxazole
inability to glucuronate bilirubin; direct bilirubin is conjugated
C. phenytoin
D. gentam icin
E. prom ethazine
4. Medium chain triglycerides are used in the form ulation of dietary aids because
I. they are more rapidly absorbed than long chain fats
II. they are the usual form of anim al fats
III. they provide m ore calories, per gram , than long chain fats
A. I only
B. III only
Do not require lipase for absorption; use in cystic fibrosis
C. I and II only
D. II and III only
E. I, II, and III only
Use the profile for John Lee to answer questions 5 through 14.
5. The active ingredient in AeroBID Inhalers is
Levalbuterol (Xopenex), salm eterol (Serevent)
A. a beta agonist bronchodilator
albuterol, terbutaline, form oterol (Foradil),
B. a corticosteroid
flunisolide; dexam ethasone, betam ethasone, beclom ethasone
C. an anticholinergic
ipratropium , tiotropium (Spiriva)
D. a m ucolytic
Acetylcysteine
E. a respiratory surfactant
Survanta, Exosurf
6. The FEV 1 test is a measurem ent of
A. tidal volum e
Peak expiratory flow rate m easured with a peak flow meter
B. total lung capacity
Forced expiratory volume in one second
C. vital capacity
Red zone <400 L/m in; Yellow 400-500, Green >550
D. functional residual capacity
E. volum e of air that can be expelled in a single breath
7. W hen taking medication from a Medrol Dosepak the patient will
A. take fewer tablets each day
term ed decrem ental dosing; 6-5-4-3-2-1 but all 4 mg
B. take more tablets each day
C. take the sam e num ber of tablets each day
D. take the sam e num ber of tablets each day, but the dose per tablet increases
E. take the sam e num ber of tablets each day, but the dose per tablet decreases

Page 1

Com m unity Pharm acy Patient Record


NAM E

John K. Lee

AGE

58

ADDRESS

117 Ashewood Drive

SEX

Male

CITY

W inder

STATE

GA

ZIP 30717

RACE

White

DIAGNOSIS Asthm a, Em physem a

HT 5' 10"

ALLERGIES Aspirin

W T 176 lbs

Date

Rx No

Doctor

Drug, Form, Strength

10-5-YY

246086

Kingston

AeroBID Inhaler

10-8-YY

246512

Kingston

Am antadine Caps, 100 m g

11-5-YY

249888

Kingston

Atrovent Inhaler

11-5-YY

249889

Kingston

Medrol Dosepak

1-8-YY

256113

Kingston

Hycodan Cough Syrup

1-8-YY

256114

Kingston

Zithromax Tabs, 250 mg

2-2-YY

260872

Kingston

Norvasc Tabs, 5 mg

Am t

Directions (SIG)

Ref

2 puffs B I D

1 cap B I D

2 puffs Q I D

Card Directions

5m l q 4hr prn cough

12

2 stat, 1 q day

30

I tab daily

60

60m l

Non-Prescription Purchases
Date

Non-Prescription Product

Quant

1-6-YY

Tylenol-ES Tabs

100

1-12-YY

PeriColace Tabs

30

Reason for Purchase


General discomfort, fever
No bowel m ovements for 2 days

Pharm acist Notes and Counseling Recomm endations


Date

Pharm acist Com m ents

11-5-YY

FEV-1 Test done

1-8-YY

Patient continues to sm oke

8. Am antadine is useful in the management of


I. influenza virus A infections
II. Parkinson's Disease
III. Hem ophilus influenza infections
A. I only
B. III only
C. I and II only
D. II and III only
E. I, II, and III only

Date

Pharm acist Com m ents

Rim antadine (flum adine); Tam iflu & Relenza


Are neuram inidase inhibitors that get A&B flu
for H. Flu use am oxil, am p, Ceftin

Page 2

9. W hich has the greatest antiinflam m atory effect com pared to its mineralocorticoid effect?
A. Florinef
m ineralocorticoid (fludrocortisone); spironolactone; eplerenone (Inspra) is a
B. Deltasone
aldosterone receptor antagonist (K+ sparing diabetes issues)
C. Medrol
D. Decadron
m ethasones, cinolones, cinonides
E. Cortef
Cortisones and prednisones
10. Mr. Lee should avoid which of the following pain relievers?
A. Ecotrin
contains aspirin
B. Cafergot
C. Tylox
D. Excedrin-PM
E. Vicodin
11. Mr. Lees constipation is most likely due to
A. Zithrom ax
B. Hycodan
opiates; anticholinergics
C. Medrol
D. Tylenol-ES
E. Norvasc
not likely since he did not start taking it for a m onth after com plaint
12. The stim ulant ingredient in PeriColace Tablets is
A. Bisacodyl
traditional Dulcolax
B. Sennosides
Senokot or PeriColace Tabs
C. Magnesium hydroxide
saline, not stim ulant
D. Docusate
stool softener
E. Casanthranol
from cascara; form erly in PeriColace Caps; still available
13. W hen Mr. Lee begins to com plain of xerostom ia, this m ay be due to the
I. Am antadine Capsules
10% of patients
II. Hycodan Cough Syrup
hom atropine
xerostom ia m eans dry mouth
III. Atrovent Inhaler
ipratropium
A. I only
B. III only
Salagen is pilocarpine, 5 mg per tab, used to produce saliva
C. I and II only
Due to effects of radiation therapy damage to salivary glands
D. II and III only
E. I, II, and III only
14. If the pharm acy did not have
substitution to
A. Flum azenil
B. Flutam ide
C. Oseltam ivir Tam iflu
D. Flum adine
E. Fludrocortisone

am antadine available, the pharm acist could suggest a therapeutic


for the sam e purpose during the winter season.
RoMazicon, benzodiazepine antidote
Eulexin, antiandrogen for prostate cancer in com bo with LHRH
Caps
Analog such as leuprolide (Lupron)
rim antidine
Florinef

End of this profile; continue with the examination.


15. W hen valproic acid is prescribed for absence seizures in a patient receiving phenobarbital,
A. discontinue the phenbarbital because phenobarbital inactivates valproic acid
B. the dose of phenobarbital m ay have to be decreased because valproic acid will increase
phenobarbital blood levels
C. ethosuxrnide should be started at the sam e tim e
D. give the valproic acid in the m orning and the phenobarbital at bedtim e
E. the dose of both drugs m ay have to be higher because each drug enhances the
m etabolism of the other

Page 3

16. Some people are "slow acetylators" of isoniazid. The therapeutic effect is
A. an enhanced sensitivity to both therapy and toxicity
B. peripheral neuropathy that is resistant to pyridoxine
European/North Am erican is
C. slower metabolism and reduced therapeutic response
40-70% ; Yellow Asians 10-30%
D. slower metabolism but enhanced therapeutic response
E. slower m etabolism but no change in therapeutic response
Asians are poor m etabolizers of barbs, diazepam , m ephenytoin
17. W hich drug is a dopam ine agonist used in the treatm ent of Parkinson's Disease?
A. chlorprom azine
A & B are dopam ine antagonist and would m ake worse
B. metoclopram ide
C. tolcapone catechol-o-m ethyl transferase (COMT) inhibitor (Tasm ar); also entacopone
D. benztropine Cogentin
(Comtan)
E. brom ocriptine Parlodel
18. W hich oral benzodiazepine would be best for a patient with a history of liver disease?
A. chlordiazepoxide
B. flurazepam Dalm ane
C. diazepam
D. m idazolam Versed
E. lorazepam Ativan, has 3-OH added by liver metabolism
19. Benzodiazepine metabolism may be inhibited by concom itant use of
I. disulfiram
II. cim etidine
III. propranolol
A. I only
B. III only
C. I and II only
D. II and III only
E. I, II, and III only
20. Particle size in pharmaceutical products is usually kept small to im prove drug dissolution and oral
absorption. An example in which large particle size is used to slow dissolution is
A. griseofulvin
B. nitroprusside
C. am photericin B
D. nitrofurantoin Macrodantin; MacoBID
E. crom olyn sodium
Use the profile for Michael Drinker to answer questions 21 through 25.
21. Alcoholics are often nutritionally deficient in
I. m agnesium
II. thiam ine
W ernicke-Korsakoff Syndrom e is reduced levels of thiam ine
III. vitam in D
thiam ine dose is 100 m g daily
A. I only
B. III only
D. II and III only
C. I and II only
E. I, II, and III only
22. Approxim ately, what dose of theophylline, in m g per Kg per hour, is Mr. Drinker receiving?
(Am inophylline is 80% theophylline)
A. 80 m g per Kg per hour
2 gm Am inophylline in 1000 m ls = 2000 m g/1000 ml
B. 40 mg per Kg per hour
2 m g per m l (x) 40 m l/hr = 80 m g am inophylline/hr
C. 0.8 mg per Kg per hour
80 m g Amino/hr (x) 0.8 (80% ) = 64 m g Theo/hour
D. 0.5 m g per Kg per hour
286 lbs (div by) 2.2 lbs/kg = 130 kg
E. 0.4 mg per Kg per hour
64 m g/hr (div by) 130 kg = 0.492 (0.5) m g per kg per hr

Page 4

Institutional Pharm acy Patient Profile


Name

M ichael Drinker

Admitting
Diagnosis
Other
Illnesses
Allergies

Room 505-A

Pneumonia, Respiratory Distress

Hypertension
Long Standing Alcoholism
Sulfonamides, Penicillin

Physician Willis

Service M edicine

Age 44

Height 6' 2"

Sex M ale

W eight 286 lbs

Race White

Religion Protestant

Admitted 03-30-YY

Insurance Self pay

M edication Orders
Start

M edication

Dose Instructions

Start

M edication

Dose Instructions

3-30

Gentamicin Injection

Pharmacy to dose

3-30

HCTZ Tabs

50 mg q day

3-30

Primaxin Injection

500 mg q 6 hrs

4-01

Antabuse Tabs

250 mg B I D

3-30

Vasotec Tabs

10 mg B I D

3-30

Cordarone Tabs

600 mg B I D
Intravenous Fluid Orders

Start

Fluid and Additives

3-30

D-5-0.45 NaCl + KCl,


40 mEq + MVI, 10 ml /L

Flow Rate
80 mls per hour

Start
3-30

Fluid and Additives

Flow Rate

0.45 NaCl, 1000 mls +


Aminophylline, 2 gm

40 mls per hour

Significant Diagnostic Studies


Start

Diagnostic Study

Result (Normal)

Start

Diagonostic Study

Result Normal

3-30

Creatinine

1.1 mg/dl

3-30

C & S of sputum

Pending

3-31

Theophylline level

18 mcg/ml

3-31

Potassium

4.6 mEq/L

Date

Pharmacist M onitoring Notes Date

3-30

Gentamicin dose is 4 mg/kg/day in 2 doses

3-30

Check theophylline level when ready

Pharmacist M onitoring Notes

23. The nurse caring for M r. Drinker wants to know if his m edications can be sent up in dosage forms
for IV injection. For the Vasotec, you reply that
A. No, there is no parenteral form of Vasotec
B. No, the parenteral form of Vasotec is for IM adm inistration only
C. Yes, and the dose will not need to be changed
D. Yes, but the dose will need to be increased
E. Yes, but the dose will need to be decreased
PO absorption is only about 60%
24. The parenteral diuretic most sim ilar to hydrochlorothiazide is
A. Lasix
B. Diuril
C. Mannitol
D. Diam ox acetazolam ide
E. Edecrin ethacrynic acid, loop diuretic; no sulfur in structure

Page 5

25. Gentam icin Injection is supplied in vials with a concentration of 80 m g per 2 m ls. How many
m illiliters of the injection must be used to prepare each dose for Mr. Drinker?
A. 26 m l
B. 13 m l
286 lbs (div by) 2.2 lb/kg = 130 kg (x) 4 mg/kg/day = 520 m g/day
C. 9.5 m l
D. 6.5 m l
520 m g/day (div by) 2 doses/day = 260 m g/dose (div by) 80 m g/2 m l = 6.5 ml
E. 3.5 m l
26. W hen M r. Drinker is discharged, the Cordarone
am IODarone iodine allergy, problems
A. dose will need to be increased
Class I, II, III and IV antiarrhythmic
B. dose will need to be decreased
Possibly m ost effective agent
C. will need to be discontinued
Load w 800-1600 mg/day for 1-3 weeks
D. can only be adm inistered in a physicians office
then 600 - 800 mg/day x 1 m onth
E. will require co-adm inistration of levothyroxine
then 400 mg per day; ADRs dose related
Pulm onary rare at <300 mg per day; 15% above
End of this profile; continue with the examination.
27. Dopamine is useful in the managem ent of cardiogenic shock because
A. it produces dose-dependent increases in cardiac output and renal perfusion
B. it cannot cross the blood-brain barrier and cause CNS effects
C. it has no effects on either alpha or beta receptors
1-3 m cg/kg/min dopamine
D. it will not increase blood pressure
2-8 m cg/kg/m in beta-1 receptors
E. it is long acting after single doses
7-10 m cg/kg/m in alpha receptors
28. Drugs m ost likely to bind to cholestyram ine are
I. alkaloids
II. acids
Acidic drugs as described in com pounding passout
III. steroids
cholesterol is a steroid
A. I only
B. III only
C. I and II only
D. II and III only
E. I, II, and III only
29.

Monoam ine oxidase inhibitors inhibit the m etabolism of


I. serotonin
II. norepinephrine
III. dopam ine
A. I only
B. III only
C. I and II only
D. II and III only
E. I, II and III only

30. The recom m endation for m alaria prophylaxis is 300 mg of chloroquine base twice a week. You have
Chloroquine Phosphate Tablets, 500 m g per tablet. If chloroquine phosphate is 60% chloroquine,
how m any tablets will be required for each week of therapy?
A. one
B. two
500 mg (x) 0.6 (60% ) = 300 m g chloroquine base per tab
C. three
Hence, two tablets a week
D. four
E. seven

Page 6

31.

W hich of the following agents is a specific inhibitor of Type B m onoam ine oxidase?
A. selegiline Eldepryl
Nardil (phenelzine); Tyram ine from avocados, banana
B. pargyline Eutonyl
peel, aged m eats, fermented beans, cottage cheese,
C. tranylcypromine Parnate
protein extracts, bologna, pepperoni, salam i, soy sauce,
D. isocarboxazid Marplan
Chianti wine; also issues are caffeine, ginseng, liquers,
E. tyramine
Chocolate due to phenylethylamine and fava beans due
to dopam ine

32. The hem oglobin A-1-C measures the


A. oxygenation of hem oglobin
B. gas exchange rate of hem oglobin
C. carboxyhemoglobin
D. glycosylation of hem oglobin
E. present blood sugar level

Reflects blood sugar average over past 3 months


Fructosam ine can for previous 1-3 weeks

33. Epsom salts is a com m on nam e for


A. alum inum sulfate
PO = laxative
B. m agnesium sulfate
IV = tocolytic, sedative, hypotensive; antidote is Calcium
C. calcium sulfate
TOP = reduces swelling
D. a m ixture of calcium and alum inum sulfates
E. a m ixture of alum inum and m agnesium sulfate
34. W hen taking Neoral Liquid, the patient should be counseled to do all the following EXCEPT
A. Use a glass to take the drug, not a plastic container
cyclosporin A
B. Take Echinacea as a useful dietary supplement
all others are true; Echinacea is an
C. Do not drink grapefruit juice
Im m une system stim ulant
D. Do not change to the capsule form without a physicians supervision
E. Report any change in renal function to your physician imm ediately
35. A substance that will
A. eutectic
B. effervescent
C. hum ectant
D. surfactant
E. friable

attract and retain water is called


two solids make a liquid
bubbles
glycerin, those plastic things in drug bottles
m akes it easier for water to enter
breaks up easily

36. In the Orange Book products with a B rating in the A/B system are considered to be
I. Not therapeutically equivalent to other pharm aceutically equivalent products
II. Therapeutically equivalent to other pharm aceutically equivalent products
III. Therapeutically equivalent except for narrow therapeutic index drugs
A. I only
B. III only
C. I and II only
Official title of orange book is Approved Drug Products with
D. II and III only
Therapeutic Equivalence Evaluations
E. I, II, and III only
37. Polyvinyl alcohol is used in ophthalmic
A. a viscosity increasing agent
B. a buffer
C. an antioxidant
D. a preservative
E. a drop spreading agent

preparations as
keeps the drop in the eye, no splashing
One drop per eye q 5 m ins
Pinch down on nasolacrim al duct to prevent absorption

Page 7

38. A child of 8 years is running a tem perature of 38 O centigrade when seen in the em ergency room.
W hich of the following OTC oral liquid antipyretic agents would be appropriate?
I. Tempra
II. Liquiprin
Concern of Reyes Syndrom e with use of aspirin
III. Trilisate
Tem pra and Liquiprin are both acetam inophen
A. I only
Trilisate is a salicylate and also requires a RX; so fails twice
B. III only
The Trilisate requiring an RX is often used on MPJE
C. I and II only
D. IIand III only
E. I, II and III only
39. The mother of the child with a temperature of 38 O centigrade asks what that represents in the usual
Fahrenheit method of m easuring temperature. The pharm acist calculates that it equals
A. 98.6 degrees
B. 99.4 degrees
F = (C x 1.8) + 32
F = (38 x 1.8) + 32
or 68.4 + 32 = 100.4
C. 100.0 degrees
D. 100.4 degrees
C = (F - 32)
212 F = 100 C
32 F = 0 C
-40 F = -40 C
E. 104.0 degrees
1.8
40. Antidepressants with the most significant anticholinergic effects include
I. fluoxetine
II. venlafaxine
III. am itriptyline
A. I only
B. III only
Cardiovascular side effects
C. I and II only
D. IIand III only
E. I, II and III only
41. Proper agents for use in the managem ent of aspirin overdose include
I. sodium bicarbonate
II. acetazolam ide
Need to alkalinize the urine if salicylate levels > 40 m g/dl; often
III. naloxone
need potassium to get urine alkaline and keep it there
A. I only
B. III only
Diam ox will cause urinary alkalosis but causes blood pH to fall
C. I and II only
and this makes things worse
D. II and III only
E. I,. II, and III only
42. W hich of the following dosage form s are suitable for IV adm inistration?
I. a solution m ost IV drugs
II. an emulsion fat emulsion, propofol
III. a suspension no, particles cause embolism
A. I only
B. III only
C. I and II only
D. II and III only
E. I, II and III only
43.

W hich form s of mesalam ine are suitable for oral adm inistration?
I. capsules
II. tablets
Caps and tabs m ust be enteric coated, suspension is for rectal enema
III. suspension
A. I only
B. III only
D. II and III only
C. I and II only
E. I, II, and III only

Page 8

44.

How m any 2.25 gm tablets of Na Cl will be needed to prepare two liters of norm al saline?
A. 1
B. 2
NSS = 0.9% or 0.9 gm per 100 m l (x) 10 = 9 gm per liter (x) 2 = 18 gm needed
C. 3
D. 4
18 gm (div by) 2.25 gm per tab = 8 tablets
E. 8

45.

W hich of the following symptoms indicates a serious side effect of carbam azepine toxicity?
A. hyperactivity
B. insom nia
Do bone marrow depression; white blood cells, platelets infections
C. anxiety
and bruising
also termed m yelosuppression
D. weight gain
E. sore throat

46. Alka-Seltzer Extra Strength Antacid and Pain Reliever contains the following ingredients in each
tablet. How m any m Eq of sodium are contained in the recom m ended adult dose of two tablets? (Na
= 23, H = 1, C = 12, O = 16)
A. 13 m Eq
Aspirin
500 mg
MW Sod Bicarb = 23 + 1 + 12 + 49 = 84
B. 23 mEq
Sod bicarb 1985 m g
thus, 84 m g = 1 m eq of Na and Na bicarb
C. 47 mEq
Citric Acid
1000 m g
D. 88 m Eq
Per tablet
1985 mg (x) 2 = 3970 m g (div by) 84 mg/m Eq =
E. 112 mEq
47 m Eq
47.

The usual adult dose of activated charcoal is


A. 100 m g
B. 500 m g
Sorbitol to aid in laxative action; W ater for gastric flush
C. 2 Gram s
D. 5 Gram s
E. 25 Gram s

48. The keratoplastic concentration for salicylic acid in topical preparation is about
A. 0.1 percent
B. 2 percent
stim ulates skin growth and toughness
C. 10 percent
and above remove skin thickness
D. 20 percent
E. 40 percent
49. The classic physical sign of discoid lupus erythem atosus is
A. a bright red tongue
B. a red rash on the lower extremities
C. a red rash only on the trunk
D. a red butterfly shaped rash on the face
E. reddened hands and feet

hydralazine, procainam ide, INH


Cause lupus that goes away when
Drug is stopped

50.

The active ingredient in Kwell is


leave on for 8-14 hrs for scabies
A. lindane
30 mins for lice in hairy areas
B. clavulinic acid products
C. chromium picolinate host of uses, diabetes, weight control
D. m ethysergide Sansert; retroperitoneal fibrosis
E. pyrethrins topical lice products

51.

Diovan dim inishes blood pressure by


Atacand - candesartan; Teveten - eprosartan;
A. a direct vasodilating action
Avapro - irbesartan; Cozaar - Losarten potas;
B. stim ulating central alpha receptors
Benicar - olm esartan; Micardis - telm isartan
C. depleting sodium with a diuretic action
Diovan - valsartan
D. blocking access of angiotensin I and II to their sites of action
E. inhibiting conversion of angiotensin I to angiotensin II

Page 9

52.

Sodium biphosphate and sodium phosphate are com bined in Solu-Medrol as


A. antioxidants
B. solubilizers
C. antibacterial agents
D. a buffer system
E. ion-pair reagents

53. A solution that contains 0.1 m ole per liter of acetic acid (pKa = 4.76) is to be buffered to a pH of
4.76. The concentration of sodium acetate, in m oles per liter, m ust be
A. 5.76
B. 4.76
C. 1.0
D. 4.66
E. 0.1
54. "Take this medication with food or m ilk", is appropriate advice to a patient taking
I. Bentyl
II. Polycillin
I and II are taken before m eals
III. Feldene
A. I only
B. III only
C. I and II only
D. II and III only
E. I, II and III only
54. Levofloxacin is excreted by which of the following routes?
I. renal
II. biliary
Since drug is used for UTIs, m ust get there in quantity and in active
III. respiratory
form; only inhalation anesthetics, etc are excreted respiratory
A. I only
B. III only
C. I and II only
D. I and III only
E. I, II and III only
Questions 55 and 56 refer to the following situation. A patient with Hodgkin's Disease is taking
prednisone, prochlorperazine, procarbazine, cyclophospham ide and vincristine. The patient has been told
this regimen m ay cause nausea, vom iting, hem orrhagic cystitis, alopecia, m yelosuppression and
peripheral neuropathies.
55. If the patient experiences m yelosuppression secondary to the chem otherapy, which of the following
signs or symptoms would most likely be noted?
I. fever
II. easy bruising
III. loss of tendon reflexes due to vincristine and relatives
A. I only
B. III only
C. I and II only
D. II and III only
E. I, II and III only
56. The likelihood of developing hemorrhagic cystitis can be decreased by
A. reducing the dose of the procarbazine
B. taking the prednisone with a m eal
Mesna/Mesnex
C. taking the vineristine at bedtim e
D. drinking lots of water each day
E. perform ing a prophylactic bladder washout

Page 10

57. Mylanta-AR Acid Reducer has the sam e active ingredient as


A. Pepcid
B. Axid
C. Gaviscon
D. Tagam et
E. Carafate
58. Low doses of methotrexate are useful in the m anagem ent of
A. rheum atoid arthritis
Rheum atrex; also need folic acid to avoid anem ia
B. schistosomiasis
bilharzia; Biltricide
C. alopecia
D. dental caries
E. leukopenia
59. If a pharm acist needs to know the ingredients of Triam inicol, which of the listed references would
be most helpful?
A. The Handbook of Injectable Drugs
Review contents of each
B. The Merck Manual
C. The Merck Index
D. Harrisons Textbook of Medicine
E. Facts and Com parisons
60.

Insulin is not necessary for the movement of glucose into


A. the liver
B. skeletal muscles
CSF glucose decreases in bacterial m eningitis; CSF levels are
C. the fat cells
usually 60% of blood levels and > 50 mg/dl
D. the brain
E. cardiac muscle
Recent evidence that brain also produces insulin > ? Alzheimers

61.

A recent prosthetic heart valve replacem ent patient diagnosed with ORSA should receive
I. Vancom ycin
II. Rifam pin
Vanco is essential
III. Gentam icin
Rifam pin is suggested in m ost texts
A. I only
Gentamicin for first 2 weeks to provide Gm negative coverage
B. III only
C. I and II only
D. II and III only
E. I, II and III only

62.

Home pregnancy tests detect the presence of


A. progesterone
B. estradiol
C. chorionic gonadotropin
D. prolactin
E. leutenizing horm one == ovulation prediction tests

63. Sodium polystyrene sulfonate resin is primarily used to treat


A. hyperchlorhydria
B. hyperkalem ia
Kayexalate, 15-30 gm per dose, in sorbitol
C. hypernatrem ia
D. hyperlipidemia
E. hyperglycem ia

Page 11

64. An enzym e product used topically to debride bed sores and sim ilar lesions is
A. lipase
B. hyaluronidase
C. collagenase
Santyl Ointm ent
D. cholinesterase
E. lipoxygenase
65. The half-life of a cephalosporin antibiotic in solution at pH 5.5 is six hours and degradation follows
first order kinetics. If two gram s of this drug are dissolved in 50 m ls of D-5-W (pH 5.5), how many
m illigrams will rem ain after 24 hrs?
A. 1,500 mg
B. 750 m g
2 gram s = 2000 m g
6 hours 1000 m g
6 hours
500 m g
C. 250 mg
D. 500 m g
6 hours 250 m g
6 hours 125 m g; so after 24 hrs = 125 m g
E. 125 m g
66. Drug A blocks the liver enzym e m etabolism of Drug B. This drug interaction m eans that
A. a patient cannot use Drug A and Drug B concom itantly
B. Drug A and Drug B must be given at different tim es of the day
C. the dose of Drug A should be reduced
D. the dose of Drug B should be reduced
E. the dose of Drug B should be increased
67. To advertise as a remedy for gas associated with heartburn, OTC products must contain antacids
and sim ethicone. W hich of the following products meets these requirem ents?
I. Alka-Seltzer
II. Rolaids
AlkaSeltzer is ASA + Na bicarb + citric acid; AlkaSeltzer Gas Relief only
III. Di-Gel
has sim ethicone in it
A. I only
B. III only
Rolaids = Ca carbonate + Mg hydroxide
C. I and II only
D. II and III only
DiGel is sim ethicone, Ca Carbonate + Mg Hydroxide
E. I, II and III only
68. Fifty healthy volunteers received single 80 m g doses, orally, of Lasix or a generic product and, after
an appropriate washout period, received the other drug. The results were:

Area under the curve (from zero to infinity)


Urinary furosem ide (from zero to 24 hours)
C max (microgram s/Liter)
T max (hours)

Lasix
Generic Furosemide
6600
3960
22 m g
12 m g
2250
1440
1.2
1.8

True statements derived from this data include which of the following:
DRAW GRAPH
I. The absolute bioavailability of the generic product is 15 percent
II. The relative bioavailability of the generic product is 60 percent
III. The generic product is absorbed less rapidly and less extensively than Lasix
A. I only
B. III only
No IV dosing data so cannot say if I true or not
C. I and II only
D. II and III only
E. I, II, and III only
69. The test design described above is term ed
A. crossover
B. double blind
D. regression analysis
C. placebo controlled
E. investigator blind

Page 12

70. W hich of the iron salts listed below has the highest percentage of elemental iron?
A. ferrous fum arate = 33%
B. ferrous sulfate
= 20% if with water; 30% if exsiccated
C. ferrous gluconate = 12%
D. ferrous lactate
= 24%
IV iron is Iron Dextran (InFED, DexFerrum )
E. ferrous citrate decahydrate = 23%
iron sucrose is Venofer; Sod ferric gluconate
Com plex is ferrlecit all are low iron percents
71. The action of procainam ide is m ost similar to that of
A. lidocaine
B. phenytoin
W ashington Manual to review all antiarrhythm ic drugs
C. verapam il
D. quinidine
E. propranolol
72. W hich auxiliary label should be placed on a container of Triam inic Drops Concentrate?
I. For the eye
II. For the ear
III. Take by mouth
Despite nam e, only contains pseudoephedrine
A. I only
B. III only
C. I and II only
D. II and III only
E. I, II and III only
Henry Grover
124 Maple Ave, CITY

Age 42
06-10-YY

RX Hydrocortisone
0.5%
Lidocaine
2%
Aquaphor qs ad
30 gm s
SIG: apply to hives TID
Refills 0
Label Yes

R. Derm a, M.D.
AD 2731532

73. You have Hydrocortisone Ointm ent, 1% and Lidocaine


Ointm ent, 5% as sources for ingredients for the
prescription to the left. How m any gram s of Aquaphor
will be required?
A. 10 gram s
B. 6 grams
Do by alligation and by
C. 3 gram s
standard m ath
D. 1 gram
E. 0.1 gram

74. W hich of the following statements is true concerning the relative potency of topical dosage forms of
hydrocortisone 1.0 percent?
A. the potency of all forms is the sam e
B. the cream is the m ost potent
C. the lotion is the m ost potent
D. the ointm ent is the m ost potent based on penetration into the skin
E. the solution is the m ost potent
75.

Calcium carbonate containing antacids often result in acid rebound because


I. calcium stim ulates the release of gastrin
II. calcium carbonate antacids are solutions
III. the carbonate ion stim ulates the proton pum p to secrete acid
A. I only
B. III only
Gastrin, histam ine and acetylcholine are acid stim ulants
C. I and II only
Pump stimulation is through cyclic AMP
D. II and III only
E. I, II, and III only

Page 13

76. W hich of the following statem ents is NOT true of theophylline?


A. it relaxes the sm ooth muscles of the respiratory tract
B. it is chem ically related to caffeine
C. it increases the contractile force of the heart
D. it stim ulates the production of gastric secretions
E. it causes depression of the central nervous system
77. A uricosuric drug
A. prom otes reabsorption of uric acid in the kidney
B. prom otes excretion of uric acid in the kidney
C. blocks excretion of uric acid in the kidney
D. causes decreased production of uric acid
E. causes increased production of uric acid
78.

causes stim ulation

Probenecid - Benem id with uric


acid and penicillins
Allopurinol; colchicine

Constipation can be a comm on side effect of the drug classes listed below EXCEPT
A. cholinesterase inhibitors
increase effect or level of acetylcholine, Alzheim ers drugs
B. antidepressants
C. antipsychotics
Rest tend to be constipating
D. opiates
E. H 1 antihistam ines

79. W hat is the purpose of the pram oxine found in se veral form ulations of Neosporin?
A. it provides additional activity against Staphylococcus aureus
B. it provides activity against fungal infections
C. it provides a product that has double strengths of ingredients
D. it provides additional relief due to hydrocortisone
E. it provides a local anesthetic for pain relief
found in m any OTC products
80. W hich route of administration is used for routine doses of insulin?
A. intram uscular injection
B. intravenous bolus
C. intravenous infusion
D. subcutaneous injection
E. Intraderm al injection
81.

A drug product that has reached its expiration date has


A. degraded to less than 95% of labeled potency
B. degraded to less than 90% of labeled potency
C. degraded to less than 50% of labeled potency
D. begun to degrade below 100% of label potency
E. degraded to less than the USP required potency

82. Drug A has one com partm ent, first-order pharmacokinetics. An IV bolus of 100 m g is given; the Vd
is 20 L. The plasm a concentration at tim e zero will be
A. 5 mg per mL
B. 5 m cg per m L
Vd = Dose
20 L = 100 mg
Co = 100 m g = 5 mg/L
C. 20 mg per Liter
Co
Co
20 L
(mcg/ml)
D. 2 mcg per mL
E. 2 m g per m L
83. Demerol has been ordered as an analgesic to a patient receiving Nardil.
I. The pharm acist should fill the prescription, no problem exists
II. This is alright as long as the Dem erol and Nardil are given orally
III. This is a dangerous drug interaction, these drugs cannot be given together
A. I only
B. III only
D. II and III only
C. I and II only
E. I, II, and III only

Page 14

84. Gestational diabetes mellitus is associated with


I. the use of sulfonylureas as the sole agent
II. a need for diet control
III. a requirem ent for insulin therapy
A. I only
B. III only
C. I and II only
D. II and III only
E. I, II, and III only
85. In a first order reaction, t
I. is a constant, independent of the dose ordered
II. can be calculated from the form ula 0.693/k
III. changes, depending upon dose adm inistered
A. I only
B. III only
D. II and III only
C. I and II only
E. I, II, and III only

zero order changes based on


Quantity present

86. Carotenoids are biochem ically converted to


A. folic acid
B. vitam in A
C. pyridoxine
D. folinic acid
E. ascorbic acid
87. Products available as over-the-counter antitussives include
I. hydrocodone
II. dextrom ethorphan
NMDA inhibitor (ala Nam enda) so used to enhance learning
III. codeine
Som e states m ay restrict C-V to RX only
A. I only
B. III only
C. I and II only
D. II and III only
E. I, II, and III only
88. The transderm al patches that are changed daily include
A. Clonidine Patches
B. Estradiol Patches
Know change tim es for all patches
C. Testaderm Patches
D. Nicotine Patches
E. Fentanyl Patches
89. HIV-1 protease inhibitors have all the following properties EXCEPT
A. inhibiting the availability of necessary enzymes for viral cell replication
B. produce an additive effect when given with reverse transcriptase inhibitors
C. may prevent emergence of viral strains resistant to reverse transcriptase inhibitors
D. rapid developm ent of resistant strains if used alone
E. are excreted in the urine unchanged
heavily m etabolized
90.

A patient requests some magnesium sulfate. The pharmacist should supply


A. MgSO 3
B. Mg(SO4)2
C. MgSO 4
also called Epsom Salt
D. Mg 2SO 4
E. Mg 3SO 4

Page 15

91. An agent often used to increase phosphate excretion in renal patients is


A. Alum inum Phosphate
Alum inum hydroxide used to lower
B. Magnesium Hydroxide
No real effect
C. Calcium Acetate
PhosLo
D. Aluminum Chloride
Used to reduce sweating
E. Sucralfate
Not an antacid
92. Medications with significant anticholinergic activity include
I. am itriptyline
II. hyoscine
Diphenhydram ine is frequently included
III. ipratropium
A. I only
B. III only
C. I and II only
D. II and III only
E. I, II, and III only
93. Possible incompatibilities of drugs m ixed in Norm al Saline include
I. Norepinephrine
II. Co-trim oxazole
see Com pounding handout for others
III. Nitroprusside
A. I only
B. III only
C. I and II only
D. II and III only
E. I, II, and III only
94. To reduce the likelihood of red man syndrome" occurring with vancom ycin, you should
A. increase the dose of'vancom ycin
B. increase the intervals between doses of vancom ycin
C. decrease the intervals between doses of vancom ycin
D. adm inister each dose of vancom ycin over a longer period of tim e
usually 90 mins
E. adm inister each dose of vancom ycin over a shorter period of tim e
95. The agent used to antagonize the effects of hypermagnesemia is
A. bism uth
B. lithium
causes are renal failure and overdose when giving Mg IV in eclam psia
C. sodium
treat with Ca gluconate, 1-2 gm over 10 m ins
D. calcium
sym ptoms are lethargy and loss of reflexes; respiratory and cardiac failure
E. phosphate
are causes of death
96. An increase in cardiac output is associated with
A. hydralazine
B. methyldopa
due to vasodilation or arterial side
C. guanethidine
D. clonidine
E. propranolol
97. W hich of the following statements about neurom uscular blocking agents are correct?
I. They facilitate tracheal intubation.
II. They produce abdom inal muscle relaxation during surgery.
III. They have no deleterious actions on respirations.
A. I only
B. III only
C. I and II only
D. II and III only
E. I, II, and III only

Page 16

98. W hich of the following best describes the recomm ended dosage of Im itrex Injection?
A. 6 m g IV every 4 hours until relief is obtained
B. 6 m g SQ as a single dose that cannot be repeated
Oral tabs 100 = 50 > 25 effect
C. 6 m g IM every 6 hours for six doses
2 hr intervals; 200 mg/day m ax
D. 6 m g SQ initially, a second dose m ay be given after at least one hour
E. 6 m g IM every day on a regular basis
99. Proper instructions for a patient who wishes to use polycarbophil for diarrhea include
I. swallow the tablets whole and follow with a full glass of water
II. chew the tablets and follow with a full glass of water
III. chew the tablets but do not drink any water
A. I only
B. III only
Constipation would be I and II
C. I and II only
Mitrolan, Fibercon, etc are trade nam es
D. II and III only
Also found in Replens, the vaginal m oisturizer
E. I, II, and III only
100. Following a dose of Im itrex one might expect to see
A. an increase in the patient's blood pressure
B. a decrease in the patient's blood pressure
C. severe bronchospasm in many patients
D. an im m ediate elevation of blood sugar
E. relief of coronary artery vasospasm
101. A patients serum potassium level must be m onitored if the patient is receiving
I. Inspra
II. Aldactone
Inspra = eplerenone; 50 m g qd is usual, BID is m ax
aldosterone
III. Dyrenium
antagonist; drug interac (reduce dose) with erythro, saquinavir,
A. I only
verapam il & fluconazole = Cyp3A4
Do not give to Type II diabetics
B. II only
with microalbum inuria; hold if K = > 5.5 or SrCreat > 2.0
C. I and II only
D. II and III only
Aldactone = spironolactone (steroid) aldosterone antag Na out; K in
E. I, II, and III only Dyrenium = triam terene = Dyazide, Maxzide
cover amiloride
102. The addition of tazobactam to piperacillin (Zosyn) is done to
A. m ake the piperacillin more water soluble
B. protect the piperacillin from the action of beta lactam ase
C. improve the in-solution stability of piperacillin
D. decrease the rate of renal excretion of piperacillin
E. reduce the rate of cytochrom e P-450 destruction of piperacillin
103. True statements concerning the use of Lovenox Injection include
30 m g subQ q 12 hrs
I. it is given only by subcutaneous injection
1 m g/kg/day (or BID)
II. the dose is not determ ined by the partial throm boplastin tim e operate on Factor Xa via
III. it is indicated to prevent clots due to venous stasis
antithrom bin III
A. I only
B. III only
Lovenox = enoxaparin; 100 m g/m l ~ 100 antiFactor Xa units per ml
C. I and II only
Innohep = tinzaparin; 20,000 antiFactor Xa units per m l
D. II and III only
Fragm in = dalteparin; 10,000 antiFactor Xa units per m l; 64 mg/ml
E. I, II, and III only

Page 17

104. W hich of the following agents might be expected to interfere with allergy testing?
I. prednisone
II. azathioprine
Exam ples of drug classes that interfere
III. fexofenadine
A. I only
B. III only
C. I and II only
D. II and III only
E. I, II, and III only
105. Agents that would be indicated in the therapy of a system ic mycosis include
I. fluconazole
II. am photericin B
III. nystatin
No system ic form , not absorbed PO or available IV
A. I only
B. III only
C. I and II only
D. II and III only
E. I, II, and III only
106. Legionnaire's Disease primarily involves the
com m unity acquired pneum onia; for patients
A. urinary tract
<40 doxycycline or m acrolide; if >60 cephalB. central nervous system
osporin or Augm entin +/ macrolide
C. gastrointestinal tract
D. respiratory tract
If hospitalized use cephalosporin + pulm onary fluoroquinolone
E. skin
Gatifloxacin = Tequin, moxifloxacin = Avelox,
107. Drugs which have their doses expressed in units include all of the following EXCEPT
A. Provera
B. Vitamin A
C. Potassium Penicillin G
D. Nystatin
E. Vitam in E
108. Proper patient instructions for the use of Domeboro Solution for soaks would include
I. soak the gauze in the Domeboro Solution and then apply the wet gauze
II. apply dry gauze to the involved area, then wet the gauze
III. pour the solution over the involved area, then apply the gauze
A. I only
B. III only
True for all soak solutions; Dom eboro = Burows Solution
C. I and II only
Burows Solution is alum inum subacetate solution
D. II and III only
Prevents gauze from sticking to the wound
E. I, II, and III only
109. A physician wants a sustained release form of lithium for a patient. You could recom m end
I. Cibalith-S
II. Eskalith
Review side effects of lithium as with too m uch sodium
III. Lithobid
Guess BID for longer acting; doses of 1800 mg daily provide
A. I only
blood levels of 0.6 to 1.2 m Eq/L as target; check levels q 2-3 m os
B. III only
C. I and II only
Diuretics inc risk of toxicity; watch renal function; category D for
D. II and III only
use in pregnancy; lithium dec thyroid levels > inc thyroid gland
E. I, II, and III only
size; Li dec renal Na reabsorption > Na depletion so maintain Na
Intake; EKG changes; false elevated W BCs

Page 18

110. Products which should not be crushed prior to adm inistration include
I. Aciphex Tablets slow release tablet rabeprazole
II. E-Mycin Tablets enteric coated tablet
III. MS-Contin Tablets slow release tablet
A. I only
B. III only
C. I and II only
D. II and III only
E. I, II, and III only
111. Agents to avoid in pregnant patients include
I. Accutane
II. Chlorpropam ide
Accutane needs sticker from MD and lim ited to 30-day supply
III. Valium
A. I only
B. III only
C. I and II only
D. II and III only
E. I, II, and III only
112. A patient has been adm itted to the em ergency room suffering a severe disulfiram -like reaction.
W hich of the following drugs may have contributed to this problem .
A. Slophyllin Liquid
B. Nyquil Night Tim e Cold Liquid
about 30% ethanol; none of others have ethanol
C. Tempra Liquid
D. PediaProfen Liquid
E. Magnesium Citrate Liquid
113. Tetracyclines
A. are considered safe in pregnancy
B. are useful in Hem ophilus influenza infections of infants all others are wrong
C. should be given with antacids to reduce stom ach upset
D. are active against Pseudomonas aeruginosa
E. are useful in the treatm ent of chlam ydial infections
114. The proper solution for storage of soft contact lenses would be
A. preserved sterile normal saline
B. a solution with thiom ersol (no longer available)
Do hard vs soft vs gas perm eable
C. an antibiotic containing solution
D. hydrogen peroxide
E. unpreserved sterile norm al saline
115. A patient is concerned that his urine has turned a reddish orange. By checking his profile, you
identify the following drugs that are likely to be involved.
I. Pyridium
II. Rim actane
May stain contact lenses; Cystex is 95 m g phenazopyridine
III. Zithrom ax
Pyridium only stains urine
A. I only
B. III only
C. I and II only
D. II and III only
E. I, II, and III only

Page 19

a.

A previously well controlled Type II diabetes m ellitus patient has noticed that his blood sugar
level has recently begun to rise. W hich of the following m ay have caused the elevated blood
sugar?
A. Septra-DS
B. Maxzide
HCTZ can damage pancreas and inc blood sugar
C. Lanoxin
D. Centrum
E. Valium

116. Adults
A.
B.
C.
D.
E.

should receive a booster dose of tetanus toxoid every


6 m onths
2 years
Tetanus antitoxin rarely used
10 years
Tetanus toxoid active imm unity
1 year
Tetanus imm une globulin passive imm unity
5 years
Clostridium tetanii; also known as lockjaw

117. You have an order for 195 m g of elem ental iron daily. Your ferrous sulfate tablets contain 325 mg
of FeSO 4 + 7 H 2O per tablet. W hat instructions should be on the label of this prescription to provide
this dose? Molecular weights: Fe = 56, S = 32, O = 16, H = 1
A. one tablet day
B. one tablet B I D
Fe = 56
X m g Fe = 56 m g Fe = 65 m g Fe per tablet
C. one tablet T I D
S = 32
325 m g FeS
278 m g FeS
D. one tablet Q I D
O4 = 64
E. two tablets T I D
H14 = 14
X tabs
=
1 tab
= 3 tabs
O7 = 112
195 mg Fe
65 m g Fe
278
118. Accurate statements concerning folic acid deficiency include
I. an adequate replacem ent dose is 1 mg per day
II. one sign of deficiency is a macrocytic anemia
III. parenteral methotrexate can be used in patients unable to take oral folic acid
A. I only
B. III only
Methotrexate is a folic acid antagonist
C. I and II only
Inc mean corpuscular volume + dec m ean corp Hgb conc
D. II and III only
Macrocytic
Hypochrom ic
E. I, II, and III only
119. Possible dietary sources for folic acid include
I. leafy green vegetables
II. beef liver
III. citrus fruits
A. I only
B. III only
C. I and II only
D. II and III only
E. I, II, and III only
120. Drugs which may reduce serum folate levels include
I. leucovorin active form
II. vitam in B 12 used together to treat pernicious anem ia; FA for blood + B12 for brain
III. phenytoin
A. I only
B. III only
C. I and II only
D. II and III only
E. I, II, and III only

Page 20

122. A patient is receiving an IV infusion of 40 m g of am inophylline per hour. The physician wants to
change to oral theophylline tablets with 85% bioavailability. W hat oral dose must the patient receive
to provide about the sam e total daily dose of theophylline?
A. 100 m g every 8 hours
B. 200 mg every 8 hours
40 m g (x) 0.8 = 32 m g Theo per hr
C. 250 m g every 8 hours
32 mg (x) 24 hrs = 768 m g Theo per day
D. 300 m g every 8 hours
768 (div by) 0.85 = 903 m g per day
E. 450 m g every 8 hours
123. Flum azenil is given in the perioperative period
A. to reverse the effects of benzodiazepines
B. for post-operative pain
C. for post-operative nausea and vom iting
D. as a skeletal muscle relaxant
E. to reverse the effect of inhalation anesthetics

Neostigm ine to reverse anticholinergics

124. The approxim ate num ber of calories in 500 m L of lntralipid 20% is
A. 100
B. 500
500 ml (x) 0.2 = 100 gram s of fat (x) 9 cals per gram = 900 cals
C. 250
D. 2000
Egg white em ulsifier = 100 cals, so + 900 = 1000 cals
E. 1000
125. W hich antagonist type can always be overcom e by increasing the dose of an agonist?
A. irreversible competitive antagonists
B. irreversible non-competitive antagonists
C. reversible com petitive antagonists
D. physiologic antagonists
E. chemical antagonists
126. W hich of the following drugs must be stored in a refrigerator?
A. Xylocaine Viscous Liquid
B. Mannitol Injection
E. Pentothal Injection
G-CSF = granulocyte colony stim ulating factor
D. Neupogen Injection
filgrastim - colony stim ulating factor for neutrophils;
E. Zantac Injection
127. Treatm ent of a upper respiratory tract infection can be stopped
A. if the patients tem perature has returned to norm al
B. if all blood cultures are negative
C. if the white blood cell count has returned to norm al
D. if the respiratory rate has returned to norm al
E. if the sputum culture is negative
128. Products that contain dextrom ethorphan include all of the following EXCEPT
A. Robitussin-PE
B. Robitussin-CF
C. Robitussin-DM
D. Robitussm Maxim um Strength Cough and Cold
E. Robitussin Pediatric Cough and Cold Formula
129. W hich drug is NOT indicated to treat a system ic AIDS related opportunistic infection?
A. Septra-DS Tablets
B. Pentam Injection
Septra is 1 st line for PC Pneum onia; Mepron is oral alternate
C. Vancocin Capsules
Pentam is IV and inhaled alternative
D. Mepron Suspension
Vanco Caps are not absorbed
E. Diflucan Tablets
Used for fungal infections

Page 21

130. A hypertensive patient is restricted to 2 gram of sodium per day. W hich of the following drug
products should be avoided?
I. K-Lyte Tablets
II. AlkaSeltzer Extra Strength Antacid and Pain Reliever
III. Fleet's PhosphoSoda Oral Laxative
A. I only
B. III only
K-Lyte is K bicarb and K citrate
C. I and II only
D. II and III only
Do Fleets Oral vs Enema story
E. I, II, and III only
131. Parkinson's Disease secondary to the use of antipsychotic drugs is due to
I. degeneration of cells within the substantia nigra
II. an increase in prolactin levels in the pituitary gland
III. blockade of dopam ine receptors within the nigrostriatal tract
A. I only
B. III only
C. I and II only
D. II and III only
E. I, II, and III only
132. Glycerin Suppositories should be stored in the refrigerator to reduce their exposure to
I. light
II. oxygen
III. hum idity
A. I only
B. III only
C. I and II only
D. II and III only
E. I, II, and III only
133. A patient with dim inished renal function should not be treated with oral
A. ferrous sulfate
B. nystatin
Use of indom ethacin to produce closure of patent ductus arteriosus
C. prednisone
Do ductus arteriosus
Prostin-VR is alprostadil
D. indomethacin
or any other NSAID
E. neom ycin
134. Probenecid m ay be com bined with ceftriaxone in the treatment of sexually transmitted diseases.
The purpose of the probenecid is
A. to decrease the excretion rate of the ceftriaxone
B. to increase the excretion rate of the ceftriaxone
C. to block the effect of penicillinase on ceftriaxone
D. to reduce allergic reactions to the ceftriaxone
E. to convert the ceftriaxone its active form
135. Release of the active ingredient from Azulfidine Entabs is determ ined by
A. tim e of adm inistration
B. volum e of water consum ed with the dose
5-ASA is released
C. presence of intestinal bacteria
D. taking the drug before meals
E. presence of lipase enzym e

Page 22

136. The first order k for piperacillin elim ination is 0.231. A patient's peak level after a dose is 40 mg/L.
If the MIC for the infecting agent is 2.5 m cg/m L, doses should be repeated every
hours to
m aintain the blood level at, or above, the MIC.
A. 3 hours
0 hrs = 40 m cg/m l
B. 6 hours
t = 0.693 = 0.693 = 3 hrs
3 hrs = 20
C. 9 hours
k
0.231
6 hrs = 10
D. 12 hours
9 hrs = 5
E. 15 hours
40 m g/L = 40 m cg/m L
12 hrs = 2.5 m cg/m l
137. A diabetic is adm itted to the emergency room in severe ketoacidosis. W hich of the following WOULD
NOT be appropriate therapy?
A. adm inistration of intravenous glargine insulin by IV drip cannot give IV
B. adm inistration of intravenous regular insulin by IV drip
necessary therapy
C. adm inistration of sodium bicarbonate by IV drip
for acidosis
D. adm inistration of potassium chloride by IV drip
Do hypokalem ia
E. adm inistration of 0.45% sodium chloride Injection by IV drip
often dehydrated
138. W hich of the drugs CANNOT be switched to IV dosing?
A. Cardizem
B. Kayexalate
C. Zantac
D. Calan
E. Erythrocin
139. W hich of the following could be classified as a herbal laxative?
A. sennosides
B. bisacodyl
C. m ineral oil
D. magnesium citrate
E. polycarbophil
Use the RX to the right to answer Questions 140 through 144.
140. The total weight of powdered ingredients will be about
A. 8 gram s
B. 38 gram s
one pint = 473 m ls
C. 90 grams
aa = 8% x 2 or 16%
D. 16 gram s
473 m ls (x) 0.16 = 75.68 gram s
E. 75 gram s

Calam ine
Zinc Oxide
Olive Oil
Lim e W ater

aa 8%
aa qs ad 1 pint

141. Correct procedure for preparation of the product will be to


A. m ix the calam ine and zinc oxide with the olive oil, and gradually add the lim e water
with constant agitation
B. m ix calam ine and lim e water, zinc oxide and olive oil, and then com bine them
C. m ix the zinc oxide with the lim e water, the calam ine with the olive oil, and then
com bine the two mixtures
D. make an em ulsion of the olive oil and the lim e water, com bine the calam ine and the
zinc oxide, then mix the emulsion and the calam ine/zinc oxide m ixture
E. com bine calam ine and zinc oxide, wet them with lim e water, then add the olive oil
142. Another nam e for Lim e W ater is
A. Calcium chloride suspension
B. Calcium hydroxide solution
C. Magnesium carbonate suspension
D. Magnesium hydroxide suspension
E. Magnesium carbonate solution

Page 23

143. The prim ary active agent in calam ine is


A. m ercuric oxide
B. ferric oxide
ZnO = 98% , Fe oxide = 2%
C. zinc oxide
D. ferrous sulfate
E. calcium carbonate
144. W hen properly prepared, the product will be a
A. white suspension
B. thick pink lotion
due to Calam ine is pink
C. thin white lotion
D. clear pink solution
E. thick white paste
145. Phenytoin side effects that are dose related include
I. gingival hyperplasia length of therapy
II. nystagm us dose
III. ataxia dose
A. I only
B. III only
C. I and II only
D. II and III only
E. I, II, and III only
146. Agents considered to be first line therapy in the management of absence seizures include
A. valproic acid
at worst second choice agent
B. trim ethadione
Ethosuximide (Zarontin) is probably drug of choice
C. phenytoin
Clonazepam (Klonopin) is useful but tolerance develops rapidly
D. diazepam
Acetazolam ide (Diam ox) for refractory cases
E. chloral hydrate
147. Antidepressants that block reuptake of both serotonin and norepinephrine include
A. phenelzine MAOI, Parnate and Nardil
B. venlafaxine Effexor
C. fluoxetine Prozac, Sarafem weekly capsule is 90 m g all ar HCl salt
D. St. Johns W ort working again
E. clozapine Clozaril
148. A severely agitated alcoholic experiencing withdrawal sym ptom s should receive
A. fentanyl no reason for analgesia
B. prochlorperazine could worsen sym ptom s due to liver issues and CNS depression
C. haloperidol best choice
D. fluoxetine no benefit
E. disulfiram would cause problem s
149. A patient receiving long term sulfasalazine therapy should also receive
A. Folic acid 1 mg a day is adequate
B. Cim etidine
C. Phenytoin
sulfonam ide action causes loss of bacteria; source for folic acid
D. Lansoprazole
lactulose can decrease bacterial activity
E. Lactulose

Page 24

150. W hich of the chemotherapeutic agents listed is highly emetogenic?


A. 5-fluorouracil low to moderate, depending on dose
B. vincristine none
C. bleom ycin none
D. cisplatin Platinol; as high as it gets
E. methotrexate low, but increases with dose
151. A patient asks for a low sodium stool softener from the laxative products. You recomm end
A. Colace sodium docusate cardiology avoids
B. Fleet's PhosphoSoda very high sodium
C. Surfak calcium docusate obstetrics avoids
D. Correctol bisacodyl
E. Senokot sennosides
152. Drixoral Cold and Flu Tablets contain
I. an analgesic acetam inophen
II. a decongestant pseudoephedrine
III. an anthhstam ine dexbrom pheniram ine
A. I only
B. III only
C. I and II only
D. II and III only
E. I, II, and III only
153. S.S. KI Drops are administered
A. orally
B. into the ears
C. topically, on to skin lesions
D. into the eyes
E. into the nose

100% = 1 gm per m l; dose is 0.3 m l


use for blocking thyroid gland in radiation issues

154. W hich of the following antacids contains calcium as the primary active ingredient?
A. Riopan magaldrate and simethicone
B. Childrens Mylanta calcium carbonate
C. Di-Gel
D. Basalgel
E. Gaviscon
155. An AccuChek device is used to measure
A. tym panic temperature
B. serum cholesterol
C. hem oglobin A-1-C
D. blood glucose
E. fructosam ine
156. Degradation of prochlorperazine edisylate injection would be recognized by
I. form ation of a precipitate
II. only by chemical assay
III. change of solution color
to yellow, true for all phenothiazines
A. I only
B. III only
C. I and II only
D. II and III only
E. I, II, and III only

Page 25

157. W hich of the following agents DOES NOT have a drug interaction with Nizoral?
A. diphenhydram ine
B. om eprazole
C. ranitidine
D. isoniazid
E. rifam pin
158. A nurse has an order to dissolve a 1 gram vial of Ancef with 1% Xylocaine Injection. She
asks if this can be done. You reply that
A. No, the drugs are incom patible and a precipitate will form
Find answer in
B. No, the drugs are incom patible and a color change will occur
Trissels
C. No, the Ancef will be inactivated even though no visial change will occur
D. Yes, this com m only done to reduce the pain of Ancef Injections
E. Yes, but a color change will occur that does not alter the Ancef efficacy
159. W hich of the following IS NOT found in Poly-Vi-Sol Tablets?
A. ferrous sulfate
B. folic acid
C. pyridoxine
D. vitam in A
E. vitam in D
160. The action of acyclovir is relatively specific for herpes sim plex infected cells because
A. the drug has no action on anything except viral m itochondria
B. the drug is applied topically directly to the infected area
Need phosphate to
C. it cannot penetrate cells that do not contain a herpes sim plex virus
form nucleotide
D. the herpes sim plex virus is needed to convert acyclovir to the active form
E. a herpes sim plex virus does not have the enzym es needed to inactivate acyclovir
161. MS-Contin is
A. an oral solution of morphine sulfate
B. a transderm al patch of morphine sulfate
C. a sustained release tablet of m orphine sulfate
D. a depot injection of morphine sulfate
E. a rectal suppository of morphine sulfate

Equianalgesic dose conversions;


usually gave reference table

162. W hich of the following agents IS NOT indicated as a treatm ent for prostate cancer?
A. leuprolide
Lupron gonadotroprin releasing horm one analog (also Zoladex),
B. goserelin = Zoladex
both inc LH and FSH and decrease testosterone production; these
C. flutam ide = Eulexin
are used in combo with Eulexin and Casodex that cause decreased
D. fluoxym esterone = Halotestin
testosterone binding
E. estram ustine = Estracyte
163. W hich of the following agents is a prodrug?
I. dipivefrin = Propine eye drops for epinephrine
II. cyclophospham ide = ifosfam ide = Ifex
III. sulindac = indom ethacin (Clinoril)
A. I only
B. III only
C. I and II only
D. II and III only
E. I, II, and III only

Mesna for bladder irritation

Page 26

164. Medications used in treating an overdose of Darvocet-N-100 would include


I. naloxone
II. acetylcysteine
III. flum azenil benzodiazepines
A. I only
B. III only
C. I and II only
D. II and III only
E. I, II, and III only
165. A positive Coombs test would indicate
A. hypothyroidism
B. increased pituitary function
C. decreased renal function
D. hem olytic anem ia
Causers are PCNs and m ethyldopa; requires transfusions
E. improved respiratory function
166. True statements concerning selegilene therapy include
Eldepryl
I. it acts as an inhibitor of type B m onam ine oxidase
only in the brain
II. severe headache can be a side effect
III. doses should be lim ited to 10 mg per day
A. I only
B. III only
Metabolized to am phetam ine and m etham phetam ine
C. I and II only
D. II and III only
E. I, II, and III only
167. Baclofen is an antispasticity agent that m im ics the action of
A. gam m a am inobutyric acid
B. serotonin
C. diazepam
D. chlorprom azine
E. dopam ine

= Lioresal

168. W hich of these drugs SHOULD NOT be shaken vigorously before administration?
A. Maalox Suspension
B. Gaviscon Suspension
C. Hum ulin-N Suspension
D. PediaProfen Suspension
E. Augm entin Suspension
169. Allopurinol is used in the treatm ent of gout due to its ability to alter the m etabolism of
I. purines
II. uric acid
Purines are adenosine and guanine in RNA and DNA
III. pyrim idines
A. I only
Pyrim idines are cytosine (RNA, DNA); uracil (RNA) and
B. III only
thym idine (DNA)
C. I and II only
D. II and III only
W atch for m etabolism vs form ation vs anabolic
E. I, II, and III only

Page 27

170. W hich of the following statem ents concerning interferons is most correct?
I. All interferons are used prim arily in the management of infectious diseases
II. Alfa interferons are prim arily used in the management of m alignancy
III. Beta interferons are prim arily used in the m anagem ent of m ultiple sclerosis
A. I only
B. III only
C. I and II only
D. II and III only
E. I, II, and III only
171. W hich of the following statements about Milk of Magnesia is correct?
I. The correct dose for use as a laxative is usually 30 mLs
II. The correct dose for use as an antacid is usually 5 m Ls
III. The active ingredient is magnesium sulfate Mag hydroxide
A. I only
B. III only
C. I and II only
D. II and III only
E. I, II, and III only
172. The best tim e of day to take a Lasix Tablet is
A. at bedtime
B. early in the m orning
C. with the evening meal
D. at least two hours rem oved from Lanoxin
E. at the sam e tim e as Milk of Magnesia
173. Constipation could be due to side effects of
A. Calan-SR
B. Lotensin
E. Synthroid
D. Trinsicon
E. Zoloft
174. Trinsicon is
A. a stool softener
B. an antihypertensive
C. a thyroid supplem ent
D. a hem atinic
E. a tranquilizer
175. W hich of the following agents would help control torsades de pointes?
A. MagOx
B. Lotensin
C. Zoloft
D. Lanoxin
E. Calan-SR
176. In a patient allergic to novocaine, which local anesthetic would be the best choice for
removing a small mole from her arm?
A. procaine
B. benzocaine
ester vs am ide
C. mepivacaine
D. lidocaine
E. tetracaine

Page 28

177. The active ingredient in Synthroid is


A. a m ixture of T-3 and T-4
B. T-3
C. T-4
D. thyroglobulin
E. T S H

Liotrix four T-4 and one T-3 by weight


Cytom el triiodothyronine
Levoxyl levothyroxine; dextrothyroxine is Choloxin
Proloid
Thyrotropin; thyroid stim ulating horm one from pituitary

178. Increased intraocular pressure


I. may result from excessive use of ocular glucocorticosteroids
II. may be reduced by topical or systemic use of carbonic anhydrase inhibitors
III. m ay be reduced by m iotics
A. I only
B. III only
C. I and II only
D. II and III only
E. I, II, and III only
Sod Chloride, 0.45%
Sod Bicarb, 8.4%
179. A physician asks if any changes are needed to m ake this IV
Pot Chloride
solution isotonic. Molecular W eights: Na = 23, Cl = 35.5,
H = 1, O = 16, K = 39
A. The solution is hypertonic; you need to rem ove 25 m L of sodium bicarbonate
B. The solution is approxim ately isotonic as is and no action is required
C. The solution is hypotonic and you need to add 75 m Eq of sodium chloride
D. The solution is hypotonic and you need to add 50 m Eq of sodium chloride
E. The solution is hypotonic and you need to add 25 m Eq of sodium chloride

1000 mL
50 mL
25 mEq

180. Purified W ater, USP can be used to


I. reconstitute antibiotics for intravenous use requires sterile water for injection
II. reconstitute alteplase for IV adm inistration m ust be reconstituted with water, not saline
III. reconstitute antibiotics for oral use
A. I only
B. III only
C. I and II only
D. II and III only
E. I, II and III only
181. W hich of the following com binations is available in a m ultidose inhaler?
A. Albuterol and dexam ethasone
B. Triamcinolone and ipratropium
C. Albuterol and ipratropium
= Com bivent
D. Terbutaline and salm eterol
E. Albuterol and salm eterol
182. W hich of the following patients is NOT a candidate for doxycycline therapy?
I. A 15 year old fem ale with acne
II. A pregnant lady with a kidney infection
III. A 6 year old child with otitis m edia
A. I only
B. III only
C. I and II only
D. II and III only
E. I, II and III only in my opinion

Page 29

Use the prescription shown to the right to answer questions 183 through 189.
183. W hat quantity of sodium cefazolin will be needed to
compound the prescription?
A. 60 m g
B. 200 m g
2% = 20 m g/ml (x) 30 m l = 600 m g
C. 0.006 gram s
D. 0.6 gram s
E. 0.02 gram s
184. Labeling on a 1 gram vial of sodium cefazolin injection provide
the instructions for dilution shown in the box to the right. To
obtain the correct am ount, the pharm acist added
m ls of diluent and then removed
m ls.
A. Added 4.6 m ls and used 3 m ls
600 m g
B. Added 1.6 m ls and used 1.6 mls
800 m g
C. Added 3.6 m ls and used 2 m ls
500 m g
D. Added 4.6 m ls and used 2.3 m ls
460 m g
E. Added 9.6 m ls and used 4.8 mls
480 m g

Sod Cefazolin
2 %
Sod chloride to isotonic QS
Sterile W ater QS AD
30 ml
SIG: 2 drops into OU QID

Volume Added Conc Resulting


1.6 m l
500 m g per ml
3.6 m l
250 m g per ml
4.6 m l
200 m g per ml
9.6 m l
100 m g per ml

185. The E value for cefazolin is 0.15, w hat quantity of sodium chloride is needed to make the final
product isotonic?
A. 75 m g
600 mg x 0.15 = 90 m g of sodium chloride equivalent
B. 750 m g
C. 18 mg
NSS = 0.9% = 9 m g/ml (x) 30 ml = 270 m g NaCl to m ake 30 m l NSS
D. 180 m g
E. 270 m g
270 m g (m inus) 90 m g NaCl from cefazolin = 180 m g NaCl required
186. The pharm acist has vials of Sodium Chloride Injection, 2.5 mEq per mL. How many m ls will be
needed to obtain the required quantity of sodium chloride? MW of Na = 23, Cl = 35.5
A. 0.55 mls
B. 1.23 m ls
Na = 23; Cl = 35.5 == 58.5 m g (m onovalent) = 1 m Eq
C. 2.46 mls
D. 3.65 m ls
58.5 m g/m Eq (x) 1.5 m Eq/m l = 146.25 m g per m l
E. 4.44 mls
180 m g (div by) 146.25 m g/m l = 1.23 m ls of solution needed
187. The prescriber asked the pharmacist to use boric acid (E value = 0.52) instead of sodium chloride
to provide additional preservation in the product. W hat quantity of boric acid should be used?
A. 93.6 m g
B. 187.2 mg
Rem em ber, you always need MORE of the other ingredient than you do of
C. 46.8 mg
NaCl. So take the amount of NaCl (180 mg) and divide by 0.52 = 344 m g.
D. 520 mg
If you had multiplied by 0.52, you would get Choice A NAPLEX will
E. 344 m g
always offer you the wrong answer.
188. The pharmacist was not certain of the stability of sodium cefazolin after the solution is prepared.
W hat reference source would be the best source of this inform ation?
A. Trissels Handbook of Injectable Drugs
B. Rem ingtons Practice of Pharm acy
C. Am erican Drug Index
D. United States Pharm acopeia
E. Merck Index

Page 30

189. The pharm acist learned that cefazolin is stable in sodium chloride solutions for 10 days if stored
in a refrigerator. The pharm acist will be using a dropper bottle that delivers 20 drops per ml. Based
on the dose ordered, what is the m aximum am ount of solution the pharm acist should prepare to
provide adequate drug and leave one extra m illiliter of solution in the dropper bottle?
A. 4 m ls
B. 8 m ls
Dose is 2 x 2 x 4 = 16 drops per day (x) 10 days = 160 drops
C. 9 m ls
D. 12 m ls
160 drops (div by) 20 drops/m l = 8 m ls + 1 extra m l = 9 m l
E. 30 mls
End of this set of questions; continue with the examination.
190. W hich of the following drugs can be taken at the sam e tim e as an antacid?
A. Tetracycline Capsules
B. Aspirin Tabs
Available in several products of aspirin + antacids/buffers
C. Cim etidine Tabs
D. Bisacodyl Tabs
All others have som e caution against concurrent use with
E. Ciprofloxacin Tabs
antacids and enteric coatings can dissolve prem aturely
191. Tardive dyskinesia is due to
A. Hypersensitive dopam ine receptors due to chronic blockade by antipsychotic drugs
B. Decreased num bers of dopamine receptors due to destruction by antipsychotic drugs
C. Increased cholinergic activity due to depletion of adrenergic receptors
D. Inhibition of acetylcholine release by antipsychotic drugs
E. Inhibition of catechol-o-m ethyl transferase by antipsychotic drugs
192. A patient experiencing tardive dyskinesia as a result of chlorprom azine therapy would exhibit
I. Tonic-clonic seizures
II. W rithing of the arm s and legs
III. Puckering of the lips and tongue
A. I only
B. III only
Tardive dyskinesia occurs at rate of 5% of patients per year
C. I and II only
D. II and III only
E. I, II and III only
193. Castille Soap is used in the preparation of
How to prepare and adm inister
A. Calam ine Linim ent
B. Soap suds enem as
Joke with 3-H and 4-F enemas
C. W ater in oil em ulsion cream s
D. Hydrophilic Ointm ent
E. Propofol Em ulsion
Diprivan, an oil-in-water emulsion
194. Accurate statements concerning the treatm ent of organophosphate poisoning include
I. Pralidoxim e removes the organophosphate from acetylcholinesterase
II. Atropine protects cholinergic receptors from high acetylcholine levels
III. Theophylline can be used as a phosphodiesterase enzym e inducer
A. I only
B. III only
Theophylline m akes worse due to blockade of sim ilar phosphodiesterase
C. I and II only
D. II and III only
E. I, II and III only

Page 31

195. A laxative product that DOES NOT contain a stool softener is


A. Fibercon
B. Doxidan = bisacodyl; Doxidan-DSC is docusate
Fibercon is polycarbophil
C. PeriColace
also Diasorb, Mitrolan, etc
D. Surfak
E. Ex-Lax Capsules
Polycarbophil also in Replens Vaginal Moisturizer
196. W hich of the following lowers gastric acid by inhibition of H +, K +-ATPase?
A. Ranitidine = Zantac, H-2 blocker
B. Sucralfate = Carafate, ulcer protectant
C. Propantheline = ProBanthine, anticholinergic
D. Alginic Acid = Gaviscon
E. Lansoprazole = Prevacid
197. Rheumatoid arthritis patients can receive
of their disease.
I. Chrysotherapy
II. Imm unosuppresant therapy
III. Tum or necrosis factor therapy
A. I only
B. III only
C. I and II only
D. II and III only
E. I, II and III only

, in addition to NSAIDs, for treatment


Gold = Ridaura, Myochrysine, Solganol
Methotrexate
Enbrel and Hum ira
Gold, rash and kidney dam age; dosing scheme
Enbrel and Hum ira inc rate of infection

198. The action of Tilade is most sim ilar to that of


Tilade = nedocrom il
A. Ipratropium = Atrovent
B. Furosem ide = Lasix
C. Tiludronate = Skelid Actonel = risedronate, Aredia = pam idronate, Didronel = etidronate
D. Crom olyn Intal and others; m ast cell stabilizer
E. Triam cinolone = Kenalog, Kenacort, Azm acort, etc
Use the profile for Martin Lawing to answer questions 199 through 213.
199. Mr. Lawing should be instructed to take his Glucotrol-XL Tablet
A. At bedtim e
B. Before breakfast
Read profile to m ake sure patient
C. In the middle of the afternoon
is not a night shift worker
D. At any tim e during the day it is convenient to do so
E. W ith the evening meal
200. If Mr. Lawings diabetes is not well controlled, his physician could consider adding
A. Actos = pioglitazone; Avandia = rosiglitazone both once daily and in com bo w sulfonylureas
B. Proglycem = diazoxide; also Hyperstat raises blood sugar
C. Hespan = hetastarch
D. Lozol
= indapam ide
E. Glucagon = body horm one to raise blood sugar
201. Univasc was chosen because
I. It protects against the side effects of Glucophage
II. It lowers blood sugar by a different m echanism than any other agent
III. It protects the kidneys from the effects of chronically elevated blood sugar
A. I only
B. III only
Moexipril food reduces absorption so give one hr before m eals
C. I and II only
usually once daily, but can give 0.5 dose BID
D. II and III only
m aintenance dose is 7.5 - 30 m g per day; reduce if also using diuretic
E. I, II and III only

Page 32

Com m unity Pharm acy Patient Pharm acy Care Record


Nam e

Martin Lawing

SSN 240-55-4587

HT

6' 2"

Address

133 Central Ave

Age

60

WT

198 lbs

Sex

Male

Race

City, State, ZIP


Allergies

Hollyton, GA 30303

Em ployer

Iodine, Codeine, Sulfas

M edical Conditions Hypertension, Type II Diabetes


Com m ents

W hite

City Power and Light

Insurance No

240554587-001

Deliver all Rxs to office


Prescription Record

Date

RX No.

Doctor

Drug Nam e and Strength

Am t

36879

345689

Mellifer

Glucotrol-XL Tabs, 5 mg

36879

345690

Mellifer

Glucophage Tabs, 500 m g

36879

345691

Mano

Univasc Tabs, 15 mg

36879

345692

Mano

Lopressor Tabs, 50 m g

180

36879

345693

Mano

Demadex Tabs, 20 mg

36578

360023

Nephro

36578

360024

36590

90

SIG

Ref

one q day

one BID

one q AM

one BID

90

one q AM

Augm entin Tabs, 500 m g

40

i QID x 10 days

Nephro

Pyridium Tabs, 200 m g

12

one QID x 3 days

362555

Nephro

Bactrim-DS Tabs

20

i BID x 10 days

36606

366005

Mano

Lipitor Tabs, 10 mg

30

one daily

36647

388921

Nephro

Casodex Tabs, 50 mg

14

one daily

180
90

Non-Prescription Purchases
Date

Product and Purpose

36570

Saw Palm etto Urinary problem s

46

Echinacea Urinary infection

36606

Garlic Capsules dec cholesterol

Date

Product and Purpose

Pharm acist M onitoring Issues and Notes


Date

Issue to be Noted

Action Taken

36879

Hgb A 1C = 6, FBS = 84

sam e m eds continued, doing well

36570

Complains of UTI symptoms

wants to self medicate; suggested see MD

36578

UTI severe enough to see MD

cautioned on use of Pyridium

36590

UTI not cleared, chg meds

suggested stop herbals

36646

UTI cleared, PSA 11 nanogm s/m l

Talk about saw palm etto use

Page 33

202. Lopressor is a beta adrenergic blocking agent that possesses


I. Cardiac selectivity
II. Intrinsic sym pathom im etic activity
Cardiac selective
III. Low lipid solubility
Atenolol
A. I only
Betaxolol
B. III only
Bisoprolol
C. I and II only
Metoprolol
D. II and III only
Acebutolol
E. I, II and III only

W ash Manual pg 76
ISA
Carteolol
Penbutolol
Pindolol

Alpha/Beta
Labetolol
Carvedilol

Acebutolol

203. As a diuretic, Dem adex is m ost similar to


A. Ethacrynic acid = Edecrin; no sulfa m olecule in structure
B. Metolazone
= Zaroxylon
C. Indapam ide
= Lozol
D. Chlorthalidone = Hygroton
E. Hydrochlorothiazide
204. The active ingredient in garlic is
will drop lipids ~ 10% , vs statins at 40% or better
A. Allicin
Destroyed by stom ach acid so enteric coated tabs make sense
B. Aloin
Oral laxative, from aloe plant
C. Parthenolide
from feverfew for migraines
D. Hypericin
with hyperforin active ingredients in St Johns W ort
E. Silym arin
active ingredient in Milk Thistle; for liver
205. The advantage of low lipid solubility, as seen in atenolol, is
I. There is less CNS depression
II. It is secreted prim arily through the kidney
III. It provides blockade of both 1 and 2 receptors
A. I only
B. III only
I = true, cannot cross blood brain barrier unless lipophilic
C. I and II only
II = true, liver does not need to convert to water soluble form
D. II and III only
III = false, blocking beta-2 would not be an advantage
E. I, II and III only
206. The pharm acist should tell Mr. Lawing that the Pyridium Tablets
A. Will color his urine red to orange
B. Must be taken before meals
Cystex Tabs, 95 mg are OTC
C. Must be stored in the refrigerator
Lim it use of drug to 2 days
D. Must be chewed, rather than swallowed
Covers painful urination sym ptom s but does
E. Enhance the activity of the Augm entin
not elim inate infection
207. Echinacea is also known as
A. Siberian Ginseng
B. St Johns W ort
C. Milk Thistle
D. Purple cone flower
E. Maidenhair tree

Eleutherococcus
Hypericum perforatum
Silybum m arianum
Echinacea angustifolia
Ginkgo biloba

208. If M r Lawing begins to take garlic for his lipid problem s


A. He should also continue the Lipitor
B. He must stop the Lipitor
C. He cannot take the garlic and Lipitor on the same day
D. The garlic will reduce the effect of the Lipitor due to a drug interaction
E. He m ust use enteric coated garlic capsules

Page 34

209. The Hgb A 1C level indicates that


A. Mr Lawing needs to increase the doses of both his antidiabetic m edications
B. Mr Lawing needs to increase the dose of only the Glucotrol-XL
C. Mr Lawing needs to increase the dose of only the Glucophage
D. Change his therapy to insulin
E. Make no change, this is a good level
210. W hich of the drugs prescribed for Mr. Lawing m ight cause Stevens-Johnson syndrom e?
I. Lopressor
II. Bactrim -DS
Also NSAIDs and barbiturates, but any drug is possible
III. Augm entin
there is no test to determine possible reaction before drug given
A. I only
B. III only
C. I and II only
D. II and III only
E. I, II and III only
211. The PSA test result indicates that
A. M r Lawing has his blood pressure under control
value should by < 2 nanogm/ml
B. Mr Lawing m ay have prostate cancer
C. Mr Lawings UTI has com pletely cleared
D. Mr Lawings UTI has not resolved
E. Mr Lawing is suffering renal dam age due to his diabetes and hypertension
212. The pharm acist wanted to talk with Mr Lawing about the use of Saw Palm etto
A. To discuss how to use Saw Palm etto in prostate cancer
B. To explain that Saw Palm etto cannot be used as a treatm ent for prostate cancer
C. To be sure Mr Lawing is taking enough Saw Palm etto
D. To review the possible drug interaction between Saw Palm etto and Bactrim -DS
E. To explain the cholesterol lowering benefits of Saw Palm etto
213. In addition to the Casodex, Dr Nephro should have ordered
Casodex = bicalultam ide
I. Tam oxifen
Nolvadex; estrogen receptor blocker, wrong thing to do
II. Granisetron
Kytril for nausea not likely to be required
III. Goserelin
Zoladex gonadotropin releasing horm one analog that binds to androgen
A. I only
receptons and is used in com bo with flutam ide (Eulexin)
B. III only
C. I and II only
D. II and III only
E. I, II and III only
End of this profile; continue with the examination.
214. W hich of the following could be used as a generic equivalent for Fortaz?
A. Ceptaz
B. Zinacef
Ceptaz has arginine; Tazidim e and Tazicef use sodium carbonate buffer
C. Duricef
that results in gas form ation when reconstituted
D. Cefipim e
E. Claforan
215. W hat is Pulm ozym e used to bring about?
Pulm ozym e = Dornase alfa which is recombinant
A. Decreased resistance of airways
hum an deoxyribonuclease
B. Bronchodilator
C. Increased absorption of inhaled medications
cystic fibrosis patients have thick lung
D. Decreased need for corticosteroid use
secretions form ed by highly polym erized
E. Decreased viscosity of m ucous in the lungs
DNA from nuclei of dead neutrophils

Page 35

216. How should Pulm ozyme be administered?


A. Topically to chest
B. Inhalation
by jet nebulizer
C. IV
D. IM
E. Orally

Refrigerate and protect from light, NMT 24 hr at


room temperature
solution comes 1 m g per ml in 2.5 ml packages
Do not m ix in nebulizer with any other drug as
m ay inactivate the Pulm ozym e

217. How should a cystic fibrosis patient's tobram ycin be administered?


A. Continuous infusion
B. IM
Pulm onary am inoglycosides are m ore effecient than systemic
C. Nebulization
ones in cystic fibrosis patients and actually im prove the disease
D. Subcutaneously
state; Pseudomonas aeruginosa is a com m on pathogen in these
E. Bolus infusion
patients
218. W hat non-medication therapy would be appropriate in a cystic fibrosis patient?
I. Sleep with head of bed elevated 45 degrees
II. Chest compression
III. Routine physical exercise
A. I only
B. III only
C. I and II only
D. II and III only
E. I, II and III only
219. Cystic fibrosis patients DO NOT need to routinely receive which of the following vaccines?
A. W hooping cough
Pertussis
B. Varicella
Chicken pox
C. Hem ophilus influenza
D. Pneum ococcal
No increased susceptibility or m orbidity in cystic patients
E. Measles
220. W hich intravenous m edication requires intensive therapeutic monitoring?
A. Lasix
B. Vasotec
D. Quinidex
oral tab, not IV
C. Nitroglycerin
E. Metoprolol
221. If amiodarone were added to this regim en, what drug would you be m ost concerned with?
A. Lasix
B. Quinidex
both agents prolong QT interval and so have added effect; am iodarone
C. Metoprolol
has an organic iodine m olecule and causes lung problems in up to 15%
D. NTG
of pateints, esp with increase dosage; alpha & beta blocker and can
E. Vasotec
cause BP drop during IV adm in; photosensitivity & thyroid issues
222. W hich one of these is an injectable used in prostate cancer?
A. Leuprolide
Lupron; inhibits gonadotropin to block steroidogenesis and lower
B. Terazosin = Hytrin
both testosterone & estrogen synthesis; also used for endom etriosis;
C. Alendronate = Fosam ax
category X for pregnancy
D. Flutam ide = Eulexin
E. Megesterol acetate = Megace, anabolic steroid that would m ake prostate cancer worse
223. If a patient with prostate problem s is com plaining of nocturia or urinary hesitancy, what would you
suggest?
A. Add leuprolide to current regim en no, causes early urinary tract obstruction
B. Increase dose of terazosin from 1 m g to 5 m g causative agent, would m ake worse
C. Begin doxazosin at 10 mg and titrate up to 20 m g cannot give in com bo with Hytrin
D. Discontinue his furosem ide no need diuresis
E. Discontinue his terazosin and start doxazosin Cardura is better at stim ulating urine flow

Page 36

224. If a patient's total phenytoin level is 15 m cg per m L, what is the free phenytoin level?
A. 15 m cg/L
B. 1.5 mcg/L
Phenytoin is 90% protein bound so ony 10% if free phenytoin
C. 7.5 m cg/m L
D. 7.5 m cg/L
10% of 15 m cg/m l = 1.5 mcg/m l or, 1.5 m g per liter
E. 1.5 m g/L
225. If 78 mLs of water is used to reconstitute an antibiotic suspension to 100 m L with a strength of 250
m g per 5 ml, how much water needs to be added to m ake the suspension 200 m g per 5 mL?
A. 53 m ls
B. 73 m ls
250 mg/5 ml = 50 m g per m l (x) 100 m l = 5000 m g drug in the bottle
C. 93 m ls
200 m g per 5 m l = 40 m g per m l
D. 103 m ls
5000 m g (div by) 40 m g per m l = 125 m l total volum e
E. 125 m ls
125 m l (m inus) 100 m l = 25 extra m ls
78 m l + 25 m l = 103 ml
226. W hat drug is proven to decrease mortality in CHF?
A. Propranolol
B. Verapamil
Benefit not seen with propranolol but with the cardioselective agents
C. Lasix
and with the alpha + beta blocking properties
D. Lisinopril
True for ACEIs and ARBs
E. Hydralazine
227. W hich nonsedating antihistam ine can be used with erythrom ycin?
A. Chlorpheniram ine
ChlorTrim eton
B. Clem astine
Tavist
C. Brom pheniram ine
Dimetane
D. Fexofenadine
Allegra
E. Tripolidine
Actidil, part of Actifed
228. W hich of the following apply to Zofran?
Odansetron
I. Blocks the 5-HT receptor
5-HT-3 specifically; gold standard antiemetic
II. Is available as an orally disintegrating tablet
Zofran-ODT
III. Is used for N/V associated with chemotherapy Especially so
A. I only
B. III only
C. I and II only
D. II and III only
E. I, II and III only
229. W hich one of these cannot be used for GERD?
A. Bentyl = dicyclom ine makes worse, because delays stom ach em ptying, antispasmodic
B. Maalox
C. Reglan
all others useful in GERD
D. Pepcid
E. Om eprazole
230. W hich drug is most equivalent to Sandim m une?
Cyclosporin; renal and tum ors
A. Sandostatin = octreotide, used to control diarrhea secondary to certain tum ors; keep in
refrigerator; causes hypo or hyperglycem ia; lowers growth horm one
B. Neoral
Neoral cannot be substituted for Sandim m une without redosing
C. Sandoglobulin = IV gam m a globulin
D. Sansert
= methysergide for m igraines; retroperitoneal fibrosis; 6 m os on, 1 m o off
E. Neo-Calglucon = Calcium gluconate

Page 37

231. Ultram works at which receptor?


A. GABA
B. Norepinephrine
specifically at the MU opiate receptor; also inhibits reuptake of
C. Opiate
Serotonin and Norepinephrine and this causes the high
D. Dopam ine
E. Serotonin
232. Topamax com es in what form ulations?
Topiram ate
A. Sprinkle capsules
B. Tablets
Very bitter taste that m akes oral liquid hard to form ulate; must
C. Suspension
withdraw slowly; inc risk of kidney stones; watch for angle
A. I only
closure glaucom a and stop if eye pain occurs; phenytoin and
B. III only
carbamazepine dec topiramate levels 40-50% ; topiram ate and
C. I and II only
valproic acid dec each other by 10-15%
D. II and III only
E. I, II and III only
blocks neuronal sodium channels and enhances GABA activity
233. You need 1:5000 benzalkonium chloride as a preservative. You have 20 m l of a 1:200 solution in
stock. How much of the 1:200 solution will you need to prepare 60 m ls of a 1:5000 solution?
A. 1.2 m l
B. 2.4 m l
1:5000 = 1 gram (1000 m g) in 5000 m l = 1 m g in 5 m l or 12 m g in 60 ml
C. 4.8 m l
D. 5.6 m l
1:200 = 1000 m g in 200 m l = 5 m g in 1 m l
X m l = 1 m l = 2.4 ml
E. 6.0 ml
12 m g
5 mg
234. W hich of these drugs
A. Cephalexin
B. Cefazolin
C. Cefpodoxim e
D. Cefadroxil
E. Ceftibuten

is
=
=
=
=
=

available as an injection?
Keflex
Ancef or Kefzol
Vantin
Duricef
Cedax

235. W hat is the longest duration oral Toradol should be used?


A. 2 days
B. 5 days
tablets and injection; avoid com bo use with other NSAIDs; can cause bleeding
C. 10 days
and m ust be stopped pre-op; avoid all herbs with antiplatelet effect
D. 12 days
E. 30 days
parenteral adm inistration m eans can take m ore before stom ach probs start
236. W hat form ulations of Depakote are available?
I. Sprinkle capsules
Depakote Sprinkle
II. Delayed release tablets
Depakote
III. Extended release tablets
Depakote-ER
A. I only
B. III only
D. II and III only
C. I and II only
E. I, II and III only
237. You have Bacitracin Ointm ent, 250 units per gram . An eye procedure calls for the use of 2.5 grams
of ointm ent 200 units of bacitracin per gram to be applied to each eye. How m uch Sterile Vaseline
would you need to add to 5 gram s of the Bacitracin Ointm ent to m ake the proper concentration?
A. 0.125 gram s
B. 1.25 gram s
250 units/gram (x) 5 gram s = 1250 units of bacitracin
C. 12.5 gram s
200 units/gram (x) 2.5 gram s (x) 2 eyes = 1000 units of bacitracin
D. 0.25 grams
X total grams
=
5 grams
= 6250 = 6.25 gm
E. 2.5 grams
1250 units of BAC
1000 units of BAC
1000
ALSO DO BY
6.25 gm (m inus) 5 gm = 1.25 gm need to add
ALLIGATION

Page 38

238. W hich of the following antiinflam m atory agents is available as an ophthalm ic solution?
A. Indom ethacin
B. Naproxen
Na diclofenac = Voltaren; Pot diclofenac = Cataflam
C. Ibuprofen
gel = Solareze for actinic keratosis; m ust avoid sunlight when using
D. Diclofenac
= Voltaren Eye Drops
E. Ketoprofen
Ocufen = flurbiprofen; Profenal = suprofen; Acular = ketoralac
239. W hich of the following statem ents concerning Glucophage is accurate?
I. Glucophage is most useful in patients with significant renal disease
II. Glucophage can be used to enhance diagnostic studies such as intravenous pyelograms
III. Glucophage can be com bined with either sulfonylureas or insulin
A. I only
B. III only
Renal disease is relative contraindication to metform in and drug m ust
C. I and II only
be stopped for 48 hrs postprocedure when patients have diagnostic
D. II and III only
studies that use iodinated radiographic contrast media because of
E. I, II and III only
competition for excretion m echanism
240. You have been asked to consult on a HIV positive patient with a positive PPD. However, the acid
fast stain is negative and his sputum culture is negative. W hat would you recom m end?
A. Start rifam pin, 600 mg and pyrazinam ide, 2 gram s daily
B. Isoniazid, 100 mg daily for 2 months, then stop
Exposed, not active TB
C. Isoniazid, 300 m g and pyridoxine, 30 m g daily
B-6 for INH peripheral neuritis
D. Streptomycin, 1 gram IM daily
E = standard anti-TB therapy
E. Isoniazid, 300 m g + rifam pin, 600 m g + pyrazinam ide, 2 gram s + etham butol, 1200 mg daily
241. W hich is a therapeutic duplication that would require contacting the prescriber?
A. Lotrel and Zestril
Lotrel = am lodipine + benazepril; Zestril = lisinopril
B. Norvasc and Lopressor
C. HCTZ and K-Dur
D. Lipitor and Lopid = caution but can do
E. Fortaz and Nebcin

(2 ACEIs)

242. W hat does the phosphate entity on Cerebyx contribute to the use of the drug product?
A. It increases the water solubility of the phenytoin m olecule
B. It increases the lipid solubility of the phenytoin m olecule
C. It increases the oral absorption of the phenytoin m olecule
150 m g of fosphenytoin
D. It increases the shelf life of the product
= 100 m g of phenytoin
E. It m akes phenytoin useful for rectal adm inistration
243. Catecholam ines, such as dopam ine, m ay turn pink when stored. This indicates dopam ine
A. Is undergoing reduction
B. Is undergoing oxidation
Also epi & norepi; Sod m etabisulfite
C. Is interacting with the preservative in the IV solution
used as antioxidant, but not in IV
D. Is being absorbed by the IV bag
fluids; also called pressor am ines
E. Is extracted the plasticizer from the IV bag
244. W hat is the antidote to dopam ine overdose due to excessively rapid adm inistration of the drug?
I. Epinephrine
alpha and beta stim ulant
II. Norepinephrine
alpha stim ulant
III. Phentolam ine
(alpha blocker; see Ques No 1)
A. I only
B. III only
Dopam ine at 1-3 m cg/kg/m in = dopam inergic and inc renal blood flow
C. I and II only
by dilation of renal arteries called RENAL dose
D. II and III only
4-10 m cg/kg/m in is beta stim ulant called CARDIAC dose
E. I, II and III only
> 10 mcg/kg/m in is alpha stim ulant called PRESSOR dose

Page 39

245. An autoim m une disorder is one in which the bodys im m une system produces autoantibodies to an
endogenous antigen. W hich of the diseases listed below IS NOT an autoim m une disorder?
A. Hashim otos thyroiditis
Glom erulonephritis
Sjogrens Syndrom e
B. Systemic lupus erythem atosis
Goodpastures Syndrom e
Throm bocytopenic purpura
C. Graves Disease
Pem phigus
Polym yositis
Myasthenia gravis
D. Rheum atoid arthritis
Hemolytic anemia
Addisons Disease
E. Cushings disease
hyperfunction of the adrenal cortex or excess iatrogenic corticosteroids
246. You would tell a patient using a Clim ara Patch to
I. Rotate sites of patch placem ent
II. Rem ove and apply a new patch once a week
III. Do not apply the patch directly to breast.
A. I only
B. III only
C. I and II only
D. II and III only
E. I, II and III only

Estradiol

247. Each of the vaccines listed com es ready for adm inistration EXCEPT
be reconstituted prior to adm inistration.
A. Pneumovax-23 = Pneumococcal vaccine
B. IPOL = Poliovirus vaccine, injectable
C. FluMist = Influenza virus vaccine
D. Recom bivas HB = Hepatitis B vaccine, recom binant
E. Meruvax II = Rubella vaccine
also true for measles, mumps and MMR

, which must

248. The antiseizure medication that CANNOT be used for the treatment of status epilepticus is
A. Lorazepam
B. Phenobarbital
C. Phenytoin
D. Carbam azepine
Does not work and no parenteral dosage form ; others possible
E. Diazepam
249. A patient is receiving 80 m g BID of a drug with 1 st order pharm acokinetics. If the dose is increased
to 80 mg QID and the AUC increases to twice that of the old dose, what phenomenon is ocurring?
A. Enterohepatic recirculation
B. A first pass effect
C. Linear kinetics
D. Increased protein binding
E. Impaired cytochrome P-450 m etabolism
250. W hich of the following therapeutic interventions is INAPPROPRIATE in a patient with hepatic
encephalopathy?
A. Increase the intake of branched chain proteins indicated
phenothiazines can be
B. Decrease the intake of arom atic am ino acids indicated
hepatotoxic
C. Initiate therapy with phenothiazine to control nausea and vom iting
D. Lactulose treatm ent; bacteria convert to acid to reduce am m onia absorption by forming NH4
E. Neomycin treatm ent; kills bacteria that produce am m onia
251. W hich can you give orally to rehydrate a person with diarrhea and replenish electrolytes?
I. GoLytely
no, profuse diarrhea
II. K-Lyte
not antidiarrheal and only contains potassium
III. Pedialyte
purpose of sale; keep dextrose conc 2% or less to avoid osm otic diarrhea
A. I only
B. III only
D. II and III only
C. I and II only
E. I, II and III only

Page 40

252. Accurate statements about transderm al nicotine patches include all of those listed EXCEPT
A. The patch should be changed daily.
B. The old patch should be rem oved before the patient showers.
How to half a
C. The patient can continue to sm oke while using the patch.
patch.
D. Patches cannot be cut in half as a m eans of decreasing the daily dose.
E. Patients with hypertension should not use the patch without physician approval.
253. A patient is receiving Pentasa and Rheumatrex as treatm ent for her irritable bowel syndrome and
rheum atoid arthritis. W hich of the following would you expect if this therapy is working?
I. An anem ia indicated by increased m ean corpuscular volume and decreased m ean
corpuscular hem oglobin concentration.
II. A decreased ESR result
III. Decreased arthralgia
Anemia is folic acid deficiency due to chronic sulfonam ide
A. I only
B. III only
ESR is basic test for autoim m une disease; fall m eans
C. I and II only
disease is im proving
D. II and III only
E. I, II and III only
Arthralgia m eans joint pain
254. Appropriate therapy for otitis media in a 6 year old child allergic to penicillin would include
I. Cipro
II. Bactrim
Cipro contraindicated in kids except in unusual circum stances; do
III. Biaxin
child with renal infection and kidney transplant possibility
A. I only
B. III only
C. I and II only
D. II and III only
E. I, II and III only
255. W hich vaccine would be INAPPROPRIATE for a patient with AIDS?
I. Hepatitis B vaccine
Needed
II. Pneum ococcal vaccine
tough choice but not required for AIDS patients
III. Varicella
Im m une globulin is indicated post exposure as treatment
A. I only
B. III only
C. I and II only
D. II and III only
E. I, II and III only
256. W ood alcohol is toxic because it is converted to
m ethanol > form aldehyde (by alcohol
A. Ethyl alcohol
dehydrogenase > form ic acid (by acetaldehye dehydrogenase)
B. Isopropyl alcohol 1. folinic acid (leucovorin), then folic acid to inc m etab of form ic acid
C. Oxalic acid
2. Fom epizole blocks alcohol dehydrogenase and is specific antidote
D. Form ic acid
3. Ethanol competes with m ethanol m etabolism by alcohol dehydase
E. Ethylene glycol
4. Hem odialysis
257. A patient who drank methanol about 30 m inutes ago could be treated with all of the following
interventions EXCEPT
A. Ipecac
For A and B patient would need to be seen early and be alert
B. Gastric lavage
C. Activated charcoal
m ethanol particles are too small so not significantly adsorbed
D. Folinic acid
E. Ethyl alcohol

Page 41

258. Mr. Stevens is com plaining that the skin area around his colostom y site is becom ing reddened and
irritated. This is most likely due to
A. Excoriation
B. Excretion of topically irritating drugs or their metabolites
Tape burns; describe both
C. A recent change in diet
ileostom y and colostomy
D. The beginning of decom position of the colostom y
E. Infection inside the colon
259. Steps that should be taken in a patient with a high thyroid level due to excess Levoxyl include
I. Initiate therapy with either propylthiouracil or m ethim azole im m ediately
II. Begin therapy with SS KI solution
III. Stop the drug im m ediately
PTU and m ethim azole block synthesis of hormones
A. I only
but do not alter activity of existing horm ones
B. III only
C. I and II only
SSKI would provide m ore iodine to make more horm ones;
D. II and III only
product is 100% KI = one gram per m l = taken orally (10 drops)
E. I, II and III only
260. W hich of the pairs of drugs given below would represent duplicate therapy?
I. Fortaz and Nebcin
II. Nebcin and Am ikin
Two am inoglycosides
III. Fortaz and Rocephin
two 3 rd generation cephalosporins
A. I only
Possible to use 1 st and 3 rd generation cephalosporin
B. III only
C. I and II only
D. II and III only
E. I, II and III only
261. W hich pair of TPN additives will precipitate if not properly added to the final product?
A. Sodium chloride and potassium phosphate
B. Potassium chloride and potassium phosphate
C. Magnesium sulfate and potassium chloride
D. Sodium acetate and potassium chloride
E. Calcium chloride and potassium phosphate
262. Betaseron can be correctly described by each of the following statem ents EXCEPT
A. The prim ary indication for this drug is treatm ent of m ultiple sclerosis
B. Betaseron is a beta-1B interferon
C. Betaseron is generically equivalent to Avonex
Avonex is beta-1A interferon
D. After mixing, the drug should be stored in a refrigerator
E. The drug must be adm inistered subcutaneously
263. Antiarrhythm ic agents that inhibit the fast sodium channels are placed in class
drugs such as
.
W ash Manual, page 169
A. Class I and include lidocaine
B. Class II and include esm olol
C. Class III and include bretylium
D. Class IV and include verapam il
E. Class V and include digoxin

and include

Page 42

264. Statement that should be included when counseling a patient on Lithium Carbonate include
I. Increased thirst is likely
II. W eight gain, due to fluid retention, is com m on
III. Dietary salt intake should be kept to a minim um
Salt intake should be normal
A. I only
B. III only
C. I and II only
D. II and III only
E. I, II and III only
265. W hich of the following needs to be monitored while on lithium therapy?
A. AST
B. ALT
no liver or renal toxicity
C. BUN
D. CBC
altered W BC levels
E. IVP
266. W hich of the follwing describes the release of nifedipine from Procardia-XL Tablets?
A. A wax matrix
Slow-K
B. Variable release beads
Micro-K
Many -XL, -XR, etc products
C. Variable release granules
K-Dur
D. Enteric coating
Enseal not long actin
E. An ALZA tablet system
also called OROS or GITS system ; tablet has hole in middle
that lets water in to create pressure to cause release by osm osis;
tabs m ust be coated to prevent leakage prior to adm inistration
267. W hich drug should not be administered if a patients apical pulse is below 60?
I. Lanoxin
II. Tenorm in
III. Dem adex
A. I only
B. III only
C. I and II only
D. II and III only
E. I, II and III only
268. W hat instructions would you give to som eone applying EMLA?
I. Apply with a cotton swab or gloves
II. Do not apply to areas of broken skin
III. Cover with an occlusive dressing after application
A. I only
B. III only
C. I and II only
D. II and III only
E. I, II and III only
269. The cause of cystic fibrosis is
A. A fungal infection of the pancreas
B. A fungal infection of the lungs
C. A recessive autosomal gene
D. A dom inant autosom al gene
E. A shortage of acetylcholinesterase

Page 43

270. W hich of the antibiotics listed IS NOT classified as antipseudom onal?


A. Levofloxacin
B. Tobram ycin
C. Ceftazidim e
D. Doxycycline
E. Imipenem
271. In what disease is TURP indicated?
A. BPH
B. UTI
C. PVD
D. GERD
E. CHF
272. W hat has been done when a patient has an ileostomy?
A. The colon has been totally rem oved
B. The ileum has been totally rem oved
C. The ileum has been cut and the proxim al end opened onto the surface of the abdom en
D. The ileum has been cut and the proxim al end attached to the sigm oid colon
E. The stom ach has been rem oved and the esophagus attached directly to the ileum
273. W hich drug would have decreased absorption due to concurrent therapy with esomeprazole?
A. Nystatin
B. Ketoconazole
C. Amphotericin B
D. Vancom ycin
E. Griseofulvin
274. In an AIDS patient receiving Retrovir and Crixivan, CMV retinitis should be treated with
I. Xalatan
II. Ganciclovir
III. Foscarnet
A. I only
B. III only
C. I and II only
D. II and III only
E. I, II and III only

Page 44

275. An HIV positive patient is allergic to sulfonam ides. Possible alternative therapy for Pneumocystis
carinii pneum onia would include
I. Atovaquone
II. Pentam idine
III. Mebendazole
A. I only
B. III only
C. I and II only
D. II and III only
E. I, II and III only
276. W hat drug regim en is given to treat endocarditis that has occurred in a patient with a prosthetic
heart valve?
A. Nafcillin plus Nystatin
B. Tetracycline plus Neom ycin
C. Am picillin plus Gentam icin
D. Metronidazole plus Amphotericin B
E. Ketoralac plus Methylprednisolone
277. Mr. Sugarm an knows he has diabetes m ellitus. He took his
blood pressure at a pharmacy and got a result of 190/110,
prom pting him to see his doctor. The elevated blood pressure
was confirm ed and a series of laboratory tests produced the
results seen in the box to the right. W hat antihypertensive
drug should be considered for initial therapy?
A. Metoprolol
B. Clonidine
C. Lisinopril
D. Hydrochlorothiazide
E. Amlodipine

Sugarman, Albert W hite Male 52 yrs


185 lbs 5 ft 10 inches
Sodium
135 mEq/L
Potassium
4.9 mEq/L
Chloride 100 mEq/L
Carbon dioxide
30 mmol/L
Glucose
110 mg/dL
BUN
30 mg/dL
Creatinine
2 mg/dL
Tech: Bela Lugosi
Physician: Danny Dracula

278. You need a 1:750 solution of Zephiran Chloride to disinfect


the pharm acy counters. Zephiran is available as a 17% concentrated solution. How m uch Zephiran
concentrate will you mix with enough water to make one gallon of the 1:750 solution you need?
A. 8 m ls
B. 15 m ls
C. 30 m ls
D. 48 mls
E. 75 mls
279. Discoid lupus erythematosus (DLE) can be com pared to systemic lupus erythematosus (SLE) by
saying that
I. Discoid lupus only involves the skin and not interior organs
II. DLE responds to topical corticosteroids whereas SLE requires systemic corticosteroids
III. Both DLE and SLE respond to treatm ent with antim alarial drugs
A. I only
B. III only
C. I and II only
D. II and III only
E. I, II and III only
280. W hat vaccine should an elderly person with diabetes get?
A. Influenza
B. Rubella
C. Rubeola
D. Hepatitis B

Page 45

E. Hepatitis A
281. W hich herb can be safely used by a patient receiving gabapentin?
A. Valerian
B. Evening prim rose oil
C. Kava kava
D. St. Johns W ort
E. Garlic
282. How do you treat a norm ocytic, normochrom ic anemia?
A. Neupogen
B. Neulasta
C. Ferrous sulfate
D. Cyanocobalam in
E. Epogen
283. W hat cannot be removed during sterilization with a 0.22m icron filter?
A. Bacteria
B. Fungi
C. Protozoa
D. Pyrogens
E. Pseudomonas
284. Mysoline is an effective anticonvulsant because it is partially m etabolized to
A. Diazepam
B. Phenobarbital
C. Valproic acid
D. Phenytoin
E. Topiram ate
285. W hich drug do you have to take with food?
I. Saquinavir
II. Am prenavir
III. Indinavir
A. I only
B. III only
C. I and II only
D. II and III only
E. I, II and III only
286. Drugs to treat hyperphosphatemia in renal failure patients include
A. Erythropoietin
B. Sodium polystyrene disulfonate
C. Calcium acetate
D. NeutraPhos
E. Calcium gluconate
287. W hat laboratory tests should be used to monitor therapy with valproic acid?
I. Liver function tests
II. Com plete blood count
III. Platelet count
A. I only
B. III only
C. I and II only
D. II and III only
E. I, II and III only

Page 46

288. W hat happens if a patient abruptly stops prednisone treatment after receiving a dose of 50 m g per
day for six months?
I. Nothing, prednisone therapy should be stopped prom ptly
II. Return of ACTH production will be delayed
III. An adrenal crisis may occur
A. I only
B. III only
C. I and II only
D. II and III only
E. I, II and III only
289. True statements concerning the use of Accolate include
I. The drug m ust be taken with food
II. Patients taking Accolate must avoid sunlight
III. Accolate cannot be used to abort an acute asthm a attack
A. I only
B. III only
C. I and II only
D. II and III only
E. I, II and III only
290. W hich of the following im m unizations m ay be adm inistered to a pregnant wom an?
A. Tetanus toxoid
B. Varicella
C. Pneum ococcal
D. Rubeola
E. Rubella
291. W hich drug can be used for Mycobacterium avium?
I. Erythrom cyin
II. Clarithrom ycin
III. Azithrom ycin
A. I only
B. III only
C. I and II only
D. II and III only
E. I, II and III only
292. An IV solution containing potassium chloride is being administered at a rate of 30 drops per minute
using IV tubing that delivers 15 drops per mL. At the end of 8 hours, the patient has received 30
m Eq of potassium chloride. W hat was the initial concentration of potassium chloride in the IV fluid?
K = 39, Cl = 36
A. 23.5%
B. 2.35%
C. 0.235%
D. 14.9%
E. 1.49%
293. If the final concentration of dextrose in a parenteral nutrition solution is to be 25% , what device
m ust be placed in the patient?
I. A Foley catheter
II. A Swan-Ganz catheter
III. A Central catheter
A. I only
B. III only
C. I and II only
D. II and III only

Page 47

E. I, II and III only


294. A pharmacy technician is checking in the order of newly received m edications. W hich ones should
be stored in the refrigerator?
I. Pulm ozym e
II. M iacalcin
III. Neupogen
A. I only
B. III only
C. I and II only
D. II and III only
E. I, II and III only
295. Nitroglycerin is not available in which of the following dosage form s?
A. Oral liquid
B. Topical patch
C. Sublingual tablet
D. Sustained release capsule
E. Ointm ent
296. Optic nerve dam age can be due to
A. Rifam pin
B. Etham butol
C. Isoniazid
D. Streptom ycin
E. Ethionam ide
297. W hich antibiotic agents can be used for acne?
I. Tetracycline
II. Clindam ycin
III. Erythrom ycin
A. I only
B. III only
C. I and II only
D. II and III only
E. I, II and III only
298. Mr. Thomas uses a m orning com bination dose of 10 units of Regular Insulin and 30 units of NPH
Insulin. This combination is not comm ercially available. How should M r Thom as obtain the dose?
I. First, pull 10 units of Regular Insulin into a syringe and then pull 30 units of NPH
Insulin into the sam e syringe.
II. Make two injections; one with 10 units of Regular Insulin in a syringe and another with
30 units of NPH Insulin in a second syringe.
III. First, pull 30 units of NPH Insulin into a syringe and then pull 10 units of Regular
Insulin into the sam e syringe.
A. I only
B. III only
C. I and II only
D. II and III only
E. I, II and III only
299. W hich of the following sugars is indicated in the treatm ent of hepatic encephalopathy?
A. Xylitol
B. Galactose
C. Lactose
D. Lactulose
E. Fructose

Page 48

300. Rose hips are a source of


A. Methyl salicylate
B. Ascorbic Acid
C. Vitam in K
D. Vitam in E
E. Vitam in A
301. Agents useful in the management of rheum atoid arthritis include all of those listed EXCEPT
A. NSAIDS
B. Auranofin
C. Myochrysine
D. Methotrexate
E. Singulair
302. Linda has just discovered she is pregnant. W hich drugs should Linda not handle unless she is
using proper protective methods?
I. Finasteride
II. Cyclophospham ide
III. Folic acid
A. I only
B. III only
C. I and II only
D. II and III only
E. I, II and III only
303. W hich of the following beta blockers DOES NOT possess intrinsic sym pathomim etic activity?
A. Acebutolol
B. Carteolol
C. Penbutolol
D. Pindolol
E. Tim olol
304. This condition occurs in pregnant wom en and is characterized by hypertension, proteinuria, edema
and, sometim es, seizures. It is known as
A. Gestational diabetes
B. Hyperemesis gravidarium
C. Graves Disease
D. Eclam psia
E. Tocolysis
305. Each of the agents listed below can be effective in m anaging angina EXCEPT
A. Aspirin
B. Acetam inophen
C. Beta adrenergic antagonists
D. Isosorbide m ononitrate
E. Calcium channel blockers
306. The proper dose of Lovenox in the initial m anagem ent of AMI is
A. 1 m g per kg subQ q 12 hours
B. 30 mg subQ B.I.D.
C. 1 m g subQ q 12 hours
D. 30 mg IV q 12 hours
E. 1 m g IV per day
307. Medications indicated in the treatm ent of a suicide attem pt with digoxin would include all of the
following EXCEPT
A. Potassium chloride

Page 49

B.
C.
D.
E.

Lidocaine
Digoxin-FAB antibody
Ephedrine
Atropine

308. Aplastic anem ia is most likely to result from


A. Carbam azepine
B. Cyanocobalam in
C. Penicillin VK
D. Folic acid
E. Erythropoietin
309. Myasthenia gravis is an autoim m une neurom uscular disorder characterized by disruption of
neurom uscular junction receptors especially to receptors of the
I. Cholinergic nervous system
II. Adrenergic nervous system
III. Dopam inergic nervous system
A. I only
B. III only
C. I and II only
D. II and III only
E. I, II and III only
310. Drugs im plicated in Glucose-6-phosphate deficiency include all of those listed EXCEPT
A. Dapsone
B. Nitrofurantoin
C. Sulfam ethoxazole
D. Phenazopyridine
E. Chloram phenicol
311. The lifetim e dose of doxorubicin is lim ited due to the drugs
A. Renal
B. Cardiac
C. Liver
D. Central nervous system
E. Pulm onary

toxicity.

312. You told your assistant to make a 3 percent m orphine solution but som ething got misunderstood
and the result is 80 mLs of a 20 percent solution. How m uch w ater m ust you add to the 80 mLs
to reduce the concentration to 3 percent?
A. 368 mls
B. 533 mls
C. 242 mls
D. 453 mls
E. 148 mls
Use the inform ation in the box provided to answer questions 141 through 143.
313. Approxim ately how m uch atropine sulfate will you need to use
to com pound the prescription?
A. 8 grains
B. 4 grains
C. 2 grains
D. 1/8th grain
E. 1/150th grain

Atropine Sulfate
0.4 mg
Morphine Sulfate 5 mg
Acetaminophen
325 mg
DTD: 20 caps
SIG: 1-2 caps q 4 hrs prn
Dr Anodyne

314. How m uch acetam inophen will you need for the 20 capsules?

Page 50

A. 60.5 gram s
B. 6.5 gram s
C. 0.65 gram s
D. 65 gram s
E. 10 grains
315. W hat will be the total weight of ingredients in to prepare the entire quantity?
A. 6.6 gram s
B. 10.1 grains
C. 0.66 gram s
D. 1010 grains
E. 66 gram s
316. Amiodarone is used to
A. Increase urinary retention of sodium and water
B. Increase blood pressure
C. Control refractory cardiac arrhythm ias
D. Relieve the pain of angina
E. Prevent the developm ent of endocarditis
317. A patient has just received a renal transplant and one of the orders calls for Solu-Medrol, 1 gram
IV once daily for 3 days. The pharm acist should recognize that this
A. Is a test dose for the patients ability to tolerate corticosteroids
B. Is a loading dose
C. Is the therapeutic dose the patient will use for several years
D. Is the recomm ended dose to prevent early graft rejection
E. Dose is grossly inaccurate and m ust be challenged

Page 51

318. For a newly diagnosed 56 year old Type II diabetic, which of the following agents would be
appropriate initial therapy?
I. Glucagon
II. Glipizide
III. Glyburide
A. I only
B. III only
C. I and II only
D. II and III only
E. I, II and III only
319. The sam e 56 year old Type II diabetic asks what he should do if his blood sugar goes too low?
I. Drink a glass of orange juice
II. Drink a can of Coca-Cola Classic
III. Drink a glass of m ilk
A. I only
B. III only
C. I and II only
D. II and III only
E. I, II and III only
320. The prim ary dose limiting toxicity of Platinol is
A. Neurotoxicity
B. Nephrotoxicity
C. Hem atopoiesis
D. Headache
E. Em esis
321. Important issues in the administration of an IVPB dose of etoposide, 140 mg, would include
A. Monitor for tachycardia during the administration of the drug
B. Use EKG monitoring to detect possible PVCs during adm inistration
C. Monitor for a change in blood pressure (hypotension) during adm inistration
D. Infuse as rapidly as possible to m inim ize hem odynam ic effects
E. Infuse over not less than 24 hours to m inim ize hem odynam ic effects
322. A patient returns for the third course of his chem otherapy. Pre-drug adm inistration laboratory tests
reveal a calcium level of 12.4 m g/dL. This is most likely due to
A. Developm ent of metastatic sites in the bones
B. Excessive intake of dairy products to com bat nausea and vom iting
C. Dehydration due to poor nutritional status
D. The paraneoplastic syndrom e of hypercalcem ia
E. Decreased serum albumin has caused less calcium binding
323. W hich antidepressant should be avoided in a patient with cardiac arrhythm ias?
A. Sertraline
B. Fluoxetine
C. Buproprion
D. Amitriptyline
E. Venlafaxine
324. A patient is com plaining of not feeling well and concerned over recent changes in his life. This could
represent developing depression or it could represent a side effect of
A. Coum adin
B. Catapres
C. Pravachol
D. Centrum Silver
E. Tum s-EX

Page 52

325. Of the available SSRIs, which one has the least effect on the cytochrom e P 450 system?
A. Fluvoxam ine
B. Paroxetine
C. Sertraline
D. Fluoxetine
E. Citalopram
326. Mr. Jackson is a post myocardial infarction patient who is having trouble dealing with the effects
on his norm al life activities. W hich of the following agents is m ost likely to help him deal with this
depression while minim izing cardiovascular toxicities?
I. Nardil
II. Anafranil
III. Zoloft
A. I only
B. III only
C. I and II only
D. II and III only
E. I, II and III only
327. Sym ptoms likely to occur if a SSRI is abruptly discontinued include
I. Fatigue, dizziness, chills, nausea
II. Tonic-clonic seizures, com a, possible death
III. Stroke, serotonin syndrom e
A. I only
B. III only
C. I and II only
D. II and III only
E. I, II and III only
328. Thomas Maxwell is having trouble controlling his blood pressure. W hich of the m edications that he
has added in the past three m onths is most likely to be the culprit?
A. Aleve Tablets
B. Ventolin Inhaler
C. Azm acort Inhaler
D. Glucotrol-XL Tablets
E. Atrovent Inhaler
329. Joan Singletary has to walk about one kilom eter from the bus stop to her job. She commonly
experiences chest pain, due to her angina, by the tim e she is about half way through the walk.
W hat action could she take to avoid this chest pain?
A. Take one adult aspirin tablet while still on the bus.
B. Remove the nitroglycerin patch she wears until she reaches the end of the walk.
C. Chew a piece of Nicorette gum just before beginning the walk.
D. Take one sublingual tablet of Nitroglycerin, 0.4 mg, before she begins the walk.
E. W alk very rapidly so she can be at her job before the pain can begin.
330. The m ost com mon side effect of Glucophage is
A. Drowsiness
B. Diarrhea
C. Paresthesias
D. Lactic acidosis
E. Tinnitis

Page 53

331. Brenda Hodges is a 17 year old asthm atic who has been increasingly using her Ventolin Inhaler and
is now taking 1 or 2 puffs 3 or 4 tim es a day, as needed. W hat would be the best course of action
to follow at this tim e?
A. Tell Brenda she can use the Ventolin Inhaler as often as she feels necessary.
B. Stop the Ventolin Inhaler and change to Serevent Inhaler, one puff q 12 hours.
C. Have Brenda add an AzmaCort Inhaler to her regim en
D. Stop the Ventolin Inhaler and use only an Azm aCort Inhaler
E. Continue the Ventolin Inhaler but add prednisone, 10 m g po BID
332. A patients peak expiratory flow rate can be measure by use of a
I. Tidal volum e of the lungs
II. Functional capacity of the lungs
III. FEV 1
A. I only
B. III only
C. I and II only
D. II and III only
E. I, II and III only
333. An optimal FEV 1 for m ost patients is
A. 25 to 40 percent
B. 40 to 50 percent
C. 50 to 65 percent
D. 65 to 75 percent
E. > 80 percent

test.

of their personal best for this test.

334. Patients who are m onitoring their FEV 1 should do so


A. At least once a day
B. At least once a week
C. At least once a month
D. At the onset of asthm a sym ptom s
E. After the adm inistration of any inhaler
335. A patient has early signs of left ventricular failure. W hich m edication should be discontinued?
A. Ism o
B. Calan
C. Lipitor
D. HydroDiuril
E. K-Dur
336. A coronary angiogram has confirm ed that a patient with an acute MI would benefit from reperfusion
of the occluded coronary artery. W hich of the following steps would be indicated?
I. Ticlid Tablet
II. Aspirin Tablet
III. Retevase Injection
A. I only
B. III only
C. I and II only
D. II and III only
E. I, II and III only
337. W hich of these drugs is least likely to be needed in the early treatm ent of an acute M I?
A. Heparin sodium
B. Lopressor
C. Nitroglycerin
D. Cardizem
E. Morphine sulfate

Page 54

338. Once the acute phase of a m yocardial infarction has passed, which of the following m edications is
m ost likely to retard the development of CHF?
A. Zestril
B. Tenorm in
C. Isordil
D. Aspirin
E. Cordarone
339. The mechanism of action for Relenza can be stated as
A. Inhibition of influenza virus neuram inidase enzym es
B. Inhibition of Hemophilus influenza neuram inidase enzym es
C. Activation of influenza virus neuram inidase enzym es
D. Precipitation of inactivated viral particles
E. Inhibition of cyclooxygenase enzym es in influenza viruses
340. AquaMephyton is used as a source of
A. W ater soluble phenytoin
B. Vitam in K-1
C. Vitam in B-12
D. Vitam in B-6
E. W ater soluble diazepam
341. The AIDS drug that contains large am ounts of Vitam in E is
A. Lopinavir
B. Am prenavir
C. Ritonavir
D. Nevirapine
E. Indinavir
342. Lactated Ringers Injection contains 20 mg of calcium chloride dihydrate per 100 m ls of IV fluid.
How m any milliMoles of calcium are present in one liter of Lactated Ringers Injection?
Molecular weights: Ca = 40, Cl = 35.5, H = 1, O = 16
A. 1.36
B. 1.9
C. 2.66
D. 4
E. 6
343. Lactated Ringers Injection contains 20 mg of calcium chloride dihydrate per 100 m ls of IV fluid.
How m any m illiOsm oles of calcium chloride are present in one liter of Lactated Ringers Injection?
Molecular weights: Ca = 40, Cl = 35.5, H = 1, O = 16
A. 3
B. 4
C. 5
D. 6
E. 8
344. Lactated Ringers Injection contains 20 mg of calcium chloride dihydrate per 100 m ls of IV fluid.
How m any mEq of calcium are present in one liter of Lactated Ringers Injection?
Molecular weights: Ca = 40, Cl = 35.5, H = 1, O = 16
A. 1.35 mEq
B. 2.7 mEq
C. 3.6 mEq
D. 5.4 mEq
E. 7.2 m Eq

Page 55

345. Aricept is used in Alzheim ers Disease because of its ability to


A. Prevent dopam ine metabolism
B. Increase acetylcholinesterase activity
C. Block gam m a am inobutyric acid
D. Inhibit acetylcholinesterase activity
E. Increase the release of serotonin
346. Viagra inhibits the action of
A. Phosphodiesterase Type 5
B. Nitrous oxide
C. Cyclooxygenase
D. Catechol-o-m ethyl transferase
E. 3-Hydroxy-3-m ethylglutaryl Coenzym e A
347. The correct dose of tobram ycin post hemodialysis would be
A. 10% of the usual loading dose
B. 50% of the usual loading dose
C. the usual loading dose (1.0 to 1.7 mg/Kg)
D. twice the usual loading dose
348. W hich form of erythrom ycin is available for IV adm inistration?
I. Ethyl succinate
II. Stearate
III. Lactobionate
A. I only
B. III only
C. I and II only
D. II and III only
E. I, II and III only
349. A pharmacist counseling a patient on the use of Bleph-10 Ophthalm ic Drops should say
I. That the drops may cause burning when applied
II. Do not touch the dropper container to the eye
III. To wait at least 10 minutes before using a different eye drop
A. I only
B. III only
C. I and II only
D. II and III only
E. I, II and III only
350. W hat is the use of Marinol in a cancer patient?
A. Analgesic
B. Antiem etic
C. Renal protectant
D. Alkylating agent
E. Antiinfective
351. A mother says her 5 year old child has a 102 O F tem perature and wants to know what she should
do to reduce the fever?
I. Bathe the child in ice water for about two hours
II. Adm inister aspirin at a dose of 5 m g per pound of body weight every 4 hours
III. Adm inister acetam inophen at a dose of 5 m g per pound of body weight every 4 hours
A. I only
B. III only
C. I and II only
D. II and III only
E. I, II and III only

Page 56

352. Mrs Allcott is purchasing Hem atest Tablets to be used to determ ine if she has blood in her stool.
W hat should the pharm acist tell her about proper use of this test.
I. Be sure to take all the tablets just before supper
II. Do not eat any red meat for 48 hours before the test
III. Stop taking her multivitam ins plus minerals tablets for 48 hours before the test
A. I only
B. III only
C. I and II only
D. II and III only
E. I, II and III only
353. W hen changing a patient from IM m orphine sulfate to Roxanol Liquid, the prescriber should
A. Maintain the sam e dose as was given IM
B. Give half of the IM dose
C. Double the IM dose
D. Triple the IM dose
E. Give one-fourth of the IM dose
354. The indication for Indocin Injection to a neonate is
A. Prevention of rheum atoid arthrits transmission from the mother
B. Closure of a patent ductus arteriosus
C. To keep the ductus arteriosus open
D. To stop bleeding following cutting of the um bilical cord
E. Prevention of excessive bilirubin form ation
355. The decanoate salt of haloperidol, phenothiazines and tricyclic antidepressants injections is used
I. To increase the tim e between doses
II. To im prove shelf stability of the product
III. To block cytochrom e P-450 enzym e interactions
A. I only
B. III only
C. I and II only
D. II and III only
E. I, II and III only
356. Stephen W illiam s likes to surf during the sum m er but has trouble with sunburn, beginning to turn
red after only 20 minutes. If Stephen wants to stay in the water for up to four hours, he should use
a product with a sun protection factor (SPF) of at least
A. 10
B. 12
C. 15
D. 20
E. 24
Use the profile for Michael Jam es to answer questions 181 through 185.
366. The fact that the effect of Mr. Jam es anti-parkinsons drugs seem s to be dim inishing is m ost likely
due to the fact that
A. M r. Jam es is probably not taking his m edication correctly
B. Mr. Jam es has let his medications go out-of-date and lose potency
C. Mr. Jam es antihypertensive drugs are blocking the effect of the anti-parkinsons drugs
D. Loss of response to anti-parkinsons drugs is an expected change over tim e
E. Mr. Jam es is taking supplem ents of Vitam in C

Page 57

Com m unity Pharm acy Patient Pharm acy Care Record


Nam e Michael Jam es

SSN 333-55-9753

HT 6' 2"

Address 114 Belgrath Court

Age 62

W T 154 lbs

City, State, ZIP Pembroke, GA 31338

Sex Male

Race

Allergies

Em ployer Retired restaurant owner

NKDA

M edical Conditions Parkinsons Disease, Hypertension,

W hite

Insurance No 042-85-9365-01A
10-20-50% co-pay on m eds

Com m ents Anti-Parkinsons drugs effect seems to be dim inishing


Prescription Record
Date

RX No.

Doctor

Drug Nam e and Strength

3821
8

145989

Shaker

Cogentin Tabs, 1 m g

3821
8

145990

Shaker

Sinem et 25/250 Tabs

3826
2

149976

Shaker

3826
2

149977

3830
8

Am t

SIG

Ref

60

one tab BID

120

one tab QID

Eldepryl Tabs, 5 m g

60

one tab BID

Shaker

Sinem et-CR Tabs, 50-200

90

one tab q 8 hours

155004

Shaker

Requip Tabs, 0.25 m g

90

one tab TID

3834
0

157987

Shaker

Com tan Tabs, 200 m g

90

one tab TID

3799
0

159035

Pressor

Coreg Tabs, 6.25 m g

30

one tab BID

3800
4

161002

Pressor

Coreg Tabs, 12.5 m g

60

one tab BID

Non-Prescription Purchases
Date

Date

38244

Robitussin-DM Cough Syrup

38260

Vitamin C Tabs, 500 mg - 2 Gm /day

38348

Im odium Caps, 2 m g
Pharm acist M onitoring Issues and Notes

Date

Issue to be Noted

38259

Tremors inc, suggest see MD

Action Taken

Page 58

367. The action of Cogentin in the treatm ent of Parkinsons Disease can be classified as
A. Dopam inergic
B. Dopa decarboxylase inhibitor
C. Catechol-o-m ethyltransferase inhibitor
D. Monoam ine oxidase inhibitor
E. Anticholinergic
368. In order to obtain greater control over the use of levodopa in this patient, Dr. Shaker could stop the
Sinemet and prescribe Larodopa and
I. Pyridoxine
II. Haloperidol
III. Lodosyn
A. I only
B. III only
C. I and II only
D. II and III only
E. I, II and III only
369. Coreg has an advantage in the managem ent of hypertension by possessing
I. Cardioselectivity
II. Low lipid solubility
III. Both alpha and beta blocking ability
A. I only
B. III only
C. I and II only
D. II and III only
E. I, II and III only
370. W hen the Eldepryl was prescribed, the dose of the Sinemet should have
A. Rem ained unchanged
B. Slightly increased (about 10% )
C. Decreased
D. Doubled
E. Discontinued
End of this profile; continue with the examination.
371. Medications that can be com bined with captopril as therapy for hypertension include
I. triam terene
II. hydrochlorothiazide
III. propranolol
A. I only
B. III only
C. I and II only
D. II and III only
E. I, II and III only
372. You dilute 1 m l of Fungizone Injection (50 m g per 10 m l) to one liter. The concentration in the
diluted solution will be
A. 1 in 1000
B. 1 in 5000
C. 1 in 10,000
D. 1 in 100,000
E. 1 in 200,000
373. W hich of the following agents are indicated for the therapy of aerobic Gram positive infections?

Page 59

I. clindam ycin
II. azactam
III. m etronidazole
A. I only
B. III only
C. I and II only
D. II and III only
E. I, II and III only
374. RH o im mune globulin (RhoGAM, etc) is
A. given to Rh positive mothers of Rh negative babies
B. given to Rh negative mothers of Rh negative babies
C. given to Rh negative mothers of Rh positive babies
D. given to Rh positive mothers of Rh positive babies
E. given to Rh positive babies of Rh negative mothers
375. A patient with a peptic ulcer should avoid which of the following pain m edications?
I. Em pirin Com pound No. 3
II. Darvocet-N-100
III. Lortab-7
A. I only
B. III only
C. I and II only
D. II and III only
E. I, II and III only
376. Bowm ans Capsule is found in the
A. ileum
B. colon
C. brain
D. kidney
E. bladder
377. W hich of the following agents IS NOT classified as an anticonvulsant?
A. Prim idone
B. Lam otrigine
C. Gabapentin
D. Topiram ate
E. Tolcapone
378. True statements about non-nucleoside reverse transcriptase inhibitors include
I. their chem ical structure is not related to DNA or RNA
II. skin rash is a com m on side effect
III. they must be used in com bination with other antiviral agents
A. I only
B. III only
C. I and II only
D. II and III only
E. I, II and III only
379. Cystic fibrosis patients usually require therapy with som e of the agents listed EXCEPT
A. Sodium chloride supplem ents
B. Lipase supplem ents
C. Dornase alfa
D. Bronchodilators
E. Am inoglycoside antibiotics

Page 60

380. Angiotensin II receptor antagonists differ from angiotensin converting enzyme inhibitors in that
I. they are safe to use in pregnancy
II. they are available in parenteral dosage form s
III. they do not cause a bradykinin-m ediated cough
A. I only
B. III only
C. I and II only
D. II and III only
E. I, II and III only
381. Fenofibrate (TriCor) exerts its lipid lowering activity in a m anner sim ilar to
I. Atrom id-S
II. Lopid
III. Pravachol
A. I only
B. III only
C. I and II only
D. II and III only
E. I, II and III only
382. A nurse asks if she can com bine a patients doses of doxorubicin and ondansetron in the same IV
bag. The best reference source for this inform ation is
A. The Merck Index
B. The Red Book
C. Rem ingtons Practice of Pharm acy
D. The Handbook of Injectable Drugs
E. The Physicians Desk Reference
383. Products used to kill lice and their eggs on children include
I. Pronto Lice Killing Kit
II. Rid Lice Treatm ent Kit
III. R & C Lice Control Spray
A. I only
B. III only
C. I and II only
D. II and III only
E. I, II and III only
384. Active ingredients in Preparation H Hemorrhoidal Suppositories include
I. Cocoa Butter
II. Shark Liver Oil
III. Phenylephrine
A. I only
B. III only
C. I and II only
D. II and III only
E. I, II and III only
385. W hich of the following SSRI agents IS NOT indicated for the treatm ent of depression?
A. Luvox
B. Celexa
C. Prozac
D. Zoloft
E. Paxil
Use the profile for Fred Busina to answer questions 201 through 207.

Page 61

386. The pharm acist should warn Mr. Busina about the side effects of colchicine. These include all of
those listed EXCEPT
A. Nausea
B. Abdom inal cram ps
C. Tinnitus
D. Vom iting
E. Diarrhea

Page 62

Com m unity Pharm acy Patient Pharm acy Care Record


Nam e Fred Busina

SSN 333-55-9753

HT 6' 2"

Address 114 Belgrath Court

Age 62

W T 154 lbs

City, State, ZIP Pembroke, GA 31338

Sex Male

Race

Allergies

Em ployer U.S. Paper Corp

Penicillin, Sulfonam ides,

M edical Conditions Bladder Cancer, Hypertension,


Hyperlipidem ia, Gout, 2 O Bone Marrow Depression

W hite

Insurance No 042-85-9365-01A
5-10-40 copay on m eds

Com m ents Ready to get chem o for bladder cancer; will provide thru hom e infusion care
Prescription Record
Date

RX No.

Doctor

Drug Nam e and Strength

Am t

SIG

Ref

36757

345689

Pum per

Tenormin Tabs, 50 mg

90

one tab daily

38218

345690

Pum per

HCTZ Tabs, 50 mg

90

one tab daily

38218

345691

Pum per

Zyloprim Tabs, 300 mg

90

one tab daily

38218

345692

Pumper

Colchicine Tabs, 0.5 m g

30

prn for gout pain

38218

345693

Pumper

Lipitor Tabs, 40 m g

90

one tab q HS

38235

350011

Crabbe

Platinol 70 m g/M 2 IV in NS

Only on Day 2

38235

350012

Crabbe

Doxorubicin 30 mg/M 2

On day 2

38235

350013

Crabbe

Methotrexate 30 mg/M 2 IV

On days 1, 15, 22

38235

350014

Crabbe

Vinblastine 3 mg/M 2 IV

On days 2, 15, 22

38235

350015

Crabbe

Zofran 16 m g IV in NS

30 m in pre chemo

36774

350016

Crabbe

Lortab-10 Tabs

30

1 q 3-4 h prn pain

38240

351974

Crabbe

Zosyn 4.5 gm IVPB in NS

15

q 8 hrs x 5 days

38240

351975

Crabbe

Neupogen Inj, 0.48 mg/1.6 ml

10

5 m cg/kg/day

Non-Prescription Purchases
Date
38218

Date
Cystex Tabs, 95 mg for bladder infx

Pharm acist M onitoring Issues and Notes


Date

Issue to be Noted

Action Taken

38235

MVAC protocol; repeat cycle q 28 days

all chemo doses over 4 hours

38240

W BCs 15,000/m m 3 with left shift

antibiotics and Neupogen

Page 63

387. Zyloprim is useful in the management of gout because of its action as


A. A xanthine oxidase inhibiting agent
B. A purine synthetase inhibitor
C. A uricosuric agent
D. An analgesic
E. A diuretic
388. Based on inform ation on the profile, the Zosyn ordered on 09-11 should be changed to
A. Unasyn
B. Prim axin
C. Septra
D. Diflucan
E. Rifadin
389. Platinol is available in a parenteral form that contains 1 m g of drug per m L of solution. If Mr.
Busina has a body surface area of 1.9 M 2. The volum e of injection that w ill be needed for Mr.
Businas dose will be
A. 1.9 mls
B. 13 m ls
C. 29 m ls
D. 96 mls
E. 133 mls
390. During the tim e that the Platinol is being infused, Mr. Busina should also receive
A. Alum inum hydroxide suspension
B. Zinecard Injection
C. Mesna Injection
D. Epogen Injection
E. Hydrating IV fluids with mannitol
391. The volum e of Neupogen Injection required for the dose ordered for Mr. Busina is approximately
A. 0.1 mls
B. 1.1 mls
C. 1.75 mls
D. 2 m ls
E. 2.2 mls
392. A calculation error resulted in Mr. Busina receiving 100 tim es the dose of m ethotrexate that should
have been adm inistered. When inform ed, Dr Crabbe ordered the adm inistration of the antidote
A. Trim ethoprim
B. Para-am inobenzoic Acid
C. Ipecac Syrup
D. Leucovorin
E. Acetylcysteine
End of this profile; continue with the examination.
393. Dopamine is useful in the management of cardiogenic shock because
A. it produces dose-dependent increases in cardiac output and renal perfusion
B. it cannot cross the blood-brain barrier and cause CNS effects
C. it has no effects on either alpha or beta receptors
D. it will not increase blood pressure
Formula per Tablet
E. it is long acting after single doses
Aspirin
500 mg
Sodium bicarbonate
1985 mg
394. Alka-Seltzer Extra Strength Antacid and Pain Reliever contains
Citric Acid
1000 mg
the following ingredients in each tablet. How m any m Eq of
sodium are contained in the recom m ended adult dose of two

Page 64

tablets? (Na = 23, H = 1, C = 12, O = 16)


A. 13 mEq
B. 23 mEq
C. 47 mEq
D. 88 mEq
E. 112 mEq
395. W hich of the iron salts listed below has the highest percent of elemental iron?
A. ferrous fum arate
B. ferrous sulfate
C. ferrous gluconate
D. ferrous iodide
E. ferrous lactate
396. The half-life of a cephalosporin antibiotic in solution at pH 5.5 is six hours and degradation follows
first order kinetics. If two gram s of this drug are dissolved in 50 m ls of D-5-W (pH 5.5), how many
m illigrams will rem ain after 24 hrs?
A. 1,500 mg
B. 750 m g
C. 500 m g
D. 250 mg
E. 125 mg
397. The action of procainam ide is m ost similar to that of
A. lidocaine
B. phenytoin
C. verapam il
D. quinidine
E. propranolol
398. A drug has one com partm ent, first-order pharm acokinetics. An IV bolus of 100 m g is given; the Vd
is 20 L. The plasm a concentration at tim e zero will be
A. 5 mg per mL
B. 5 mcg per mL
C. 20 m g per Liter
D. 2 mcg per mL
E. 2 m g per m L
399. Dem erol has been ordered as an analgesic to a patient receiving Nardil.
I. The pharm acist should fill the prescription, no problem exists
II. This is alright as long as the Dem erol and Nardil are given orally
III. This is a dangerous drug interaction, these drugs cannot be given together
A. I only
B. III only
C. I and II only
D. II and III only
E. I, II, and III only
400. W hich of the following im m unizations confers only passive im m unity on the recipient
A. Hepatitis Im m une Globulin
B. Varicella vaccine
C. Oral polio vaccine
D. Tetanus toxoid, alum precipitated
E. Pneum ovax
401. Each of the following natural products is a useful source of Vitam in C EXCEPT
A. Broccoli

Page 65

B.
C.
D.
E.

Coffee
Orange Juice
Rose Hips
Apples

402. BCG vaccine is used to provide im munization against


A. Streptococcal infections
B. Fungal infections
C. Chickenpox
D. W hooping cough
E. Tuberculosis
403. The vaccine Prevnar is particularly indicated for which of the following patient groups?
I. Imm unocomprom ised children
II. Elderly patients
III. HIV positive patients
A. I only
B. III only
C. I and II only
D. II and III only
E. I, II, and III only
404. A mother says her child has a tem perature of 102.5 O F. The weight of the child is 47 pounds. The
m other has a bottle of Tylenol Elixir, 160 mg per 5 ml. W hat should be the dose for the child?
A. 0.5 ml every 4 hours
B. 1 m l every 4 hours
C. 10 m l every 4 hours
D. 15 ml every 4 hours
E. 5 m l every 12 hours
405. A 35 yearold female with rheum atoid arthritis patient is well controlled on Enbrel therapy. Which
of the following factors would indicate that it m ight be necessary to discontinue the Enbrel?
A. Chronic pulm onary infections
B. Reduced swelling in the fingers
C. Increased range of motion about the shoulder joint
D. Dizziness
E. A reduction in the erythrocyte sedim entation rate
406. If a patient is taking acetam inophen for chronic osteoarthritis pain and valproic acid for seizures,
which of the following param eters should you m onitor?
A. BUN and Creatinine
B. AST
C. Tidal volum e
D. Uric acid
E. W BC
407. W hich anticlotting agent works through direct inhibition of throm bin?
A. Heparin
B. Coum dadin
C. Fragm in
D. Refludan
E. Aggrenox
408. Sym byax is a com bination of
A. Ceftin and Biaxin
B. Prozac and Zyprexa
C. Prevacid and Pepcid

Page 66

D. Biaxin and Zyvox


E. Zithrom ax and Protonix
409. W hich of the following products is indicated for the management of external genital warts?
A. Benzoyl peroxide
B. Compound W
C. Condylox
D. Vitam in A and D ointm ent
E. SS KI
410. A patient has had an N/G tube placed after surgery. W hich of these medications cannot be crushed
and adm inistered through the N/G tube.
A. Dulcolax Tablets
B. Vasotec Tablets
C. Dem adex Tablets
D. Lipitor Tablets
E. Levaquin Tablets
411. Hodgkins Disease is a type of
A. Renal failure
B. Cerebrovascular disease
C. Hormone deficiency
D. Lym phom a
E. Pulm onary failure
412. A physician requests a com bination steroid and antibiotic ear drop. You suggest
I. Dom eboro Otic
II. VoSol-HC Otic
III. Cortisporin Otic
A. I only
B. III only
C. I and II only
D. II and III only
E. I, II, and III only
413. Augmentin is a com bination of
A. Clavulinic acid and am picillin
B. Clavulinic acid and ticarcillin
C. Clavulinic acid and piperacillin
D. Clavulinic acid and am oxicillin
E. Clavulinic acid and dicloxacillin
414. W hat is sodium polystyrene sulfonate used for?
A. Hypom agnesem ia
B. Hyponatrem ia
C. Hypercalcem ia
D. Hyperkalem ia
E. Hypocalcem ia
415. W hat is the antidote to benzodiazepine overdose?
A. Naloxone
B. Flum azenil
C. Naltrexone
D. Disulfiram
E. Neostigm ine
416.

Page 67

Das könnte Ihnen auch gefallen